You are on page 1of 86

PRESTORMINGTM 2022

MOCK TEST III


EXPLANATION
QUESTION NO PAGE NO
Q.1) .............................................................................................................................................. 4
Q.2) .............................................................................................................................................. 4
Q.3) .............................................................................................................................................. 5
Q.4) .............................................................................................................................................. 6
Q.5) .............................................................................................................................................. 7
Q.6) .............................................................................................................................................. 8
Q.7) ............................................................................................................................................ 10
Q.8) ............................................................................................................................................ 11
Q.9) ............................................................................................................................................ 12
Q.10) ............................................................................................................................................ 12
Q.11) ............................................................................................................................................ 14
Q.12) ............................................................................................................................................ 15
Q.13) ............................................................................................................................................ 16
Q.14) ............................................................................................................................................ 17
Q.15) ............................................................................................................................................ 17
Q.16) ............................................................................................................................................ 18
Q.17) ............................................................................................................................................ 19
Q.18) ............................................................................................................................................ 20
Q.19) ............................................................................................................................................ 20
Q.20) ............................................................................................................................................ 21
Q.21) ............................................................................................................................................ 22
Q.22) ............................................................................................................................................ 22
Q.23) ............................................................................................................................................ 23
Q.24) ............................................................................................................................................ 24
Q.25) ............................................................................................................................................ 25
Q.26) ............................................................................................................................................ 25
Q.27) ............................................................................................................................................ 26
Q.28) ............................................................................................................................................ 27
Q.29) ............................................................................................................................................ 28
Q.30) ............................................................................................................................................ 29
Q.31) ............................................................................................................................................ 29
Q.32) ............................................................................................................................................ 30
Q.33) ............................................................................................................................................ 30
Q.34) ............................................................................................................................................ 31
Q.35) ............................................................................................................................................ 32
Q.36) ............................................................................................................................................ 32
Q.37) ............................................................................................................................................ 33

1
Q.38) ............................................................................................................................................ 34
Q.39) ............................................................................................................................................ 35
Q.40) ............................................................................................................................................ 36
Q.41) ............................................................................................................................................ 36
Q.42) ............................................................................................................................................ 37
Q.43) ............................................................................................................................................ 39
Q.44) ............................................................................................................................................ 40
Q.45) ............................................................................................................................................ 40
Q.46) ............................................................................................................................................ 41
Q.47) ............................................................................................................................................ 42
Q.48) ............................................................................................................................................ 42
Q.49) ............................................................................................................................................ 43
Q.50) ............................................................................................................................................ 45
Q.51) ............................................................................................................................................ 46
Q.52) ............................................................................................................................................ 46
Q.53) ............................................................................................................................................ 47
Q.54) ............................................................................................................................................ 48
Q.55) ............................................................................................................................................ 48
Q.56) ............................................................................................................................................ 49
Q.57) ............................................................................................................................................ 50
Q.58) ............................................................................................................................................ 50
Q.59) ............................................................................................................................................ 51
Q.60) ............................................................................................................................................ 52
Q.61) ............................................................................................................................................ 53
Q.62) ............................................................................................................................................ 54
Q.63) ............................................................................................................................................ 55
Q.64) ............................................................................................................................................ 56
Q.65) ............................................................................................................................................ 57
Q.66) ............................................................................................................................................ 57
Q.67) ............................................................................................................................................ 58
Q.68) ............................................................................................................................................ 59
Q.69) ............................................................................................................................................ 60
Q.70) ............................................................................................................................................ 60
Q.71) ............................................................................................................................................ 61
Q.72) ............................................................................................................................................ 62
Q.73) ............................................................................................................................................ 63
Q.74) ............................................................................................................................................ 64
Q.75) ............................................................................................................................................ 65
Q.76) ............................................................................................................................................ 65
Q.77) ............................................................................................................................................ 66
Q.78) ............................................................................................................................................ 67
Q.79) ............................................................................................................................................ 67
2
Q.80) ............................................................................................................................................ 68
Q.81) ............................................................................................................................................ 69
Q.82) ............................................................................................................................................ 70
Q.83) ............................................................................................................................................ 70
Q.84) ............................................................................................................................................ 71
Q.85) ............................................................................................................................................ 72
Q.86) ............................................................................................................................................ 72
Q.87) ............................................................................................................................................ 72
Q.88) ............................................................................................................................................ 73
Q.89) ............................................................................................................................................ 75
Q.90) ............................................................................................................................................ 76
Q.91) ............................................................................................................................................ 77
Q.92) ............................................................................................................................................ 78
Q.93) ............................................................................................................................................ 78
Q.94) ............................................................................................................................................ 79
Q.95) ............................................................................................................................................ 80
Q.96) ............................................................................................................................................ 81
Q.97) ............................................................................................................................................ 82
Q.98) ............................................................................................................................................ 84
Q.99) ............................................................................................................................................ 84
Q.100) ............................................................................................................................................ 85

3
Q.1) Three patrol boats of border security forces are patrolling 3 rivers (River Kali, River Manas and
River Tiau) along India‘s International Borders. Which of the following borders has been patrolled via
these rivers?

1. India-Nepal border

2. India-China border

3. India-Myanmar border

Select the correct answer using the code given below.

(a) 1 and 2 only

(b) 1 and 3 only

(c) 3 only

(d) 2 and 3 only

EXPLANATION:

The Sharda River, also called Kali river and Mahakali River, originates at Kalapani in
the Himalayas at an elevation of 3,600 m (11,800 ft) in the Pithoragarh district in
Uttarakhand, India. It flows along Nepal's western border with India and has a basin area of
14,871 km2 (5,742 sq mi). Therefore, patrolling via Kali covers the India-Nepal border.
The Manas River is transboundary in the Himalayan foothills between southern Bhutan and
India. It originates in Bhutan and enters China but while entering India it enters via Bhutan
only. Therefore, patrolling via Manas covers the India-Bhutan border.
Tiau River (or Tio river) is a 159 km (99 mi) long river that forms part of the international
boundary between India and Myanmar.Therefore, patrolling via Tiau covers the India-Myanmar
border.
So, Option (b) is correct.

Q.2) Arrange the following dams in the Cauvery River basin in the order from the upstream.

1. Hemavathi dam

2. Harangi Dam

3. Krishna Raja Sagar Dam

4. Mettur Dam

Select the correct answer using the code given below.

(a) 1 – 2 – 3 – 4

(b) 1 – 2 – 4 – 3

(c) 1 – 4 – 3 – 2

(d) 2 – 4 – 1 – 3
4
EXPLANATION:

Hemavathy Dam is constructed across the river Hemavathy, an important tributary of the river
Kaveri. The dam is located in Gorur, near the city of Hassan in Karnataka.

Harangi Dam built is constructed across the river Harangi, a tributary of Kaveri located in the
Kodagu district of Karnataka.

The Krishna Raja Sagara Dam (KRS Dam) was built across the river Kaveri in the Mandya district
of Karnataka.

The Mettur Dam is one of the largest dams in India and also the largest in Tamil Nadu, located
across the river Kaveri in the Salem district of Tamil Nadu.

So, Option (a) is correct.

Q.3) Consider the following pairs of National parks and their nature.

Nature
National Parks

1. Bhitarkanika National Park - Wetland

5
2. Blackbuck National Park - Grassland

3. Bandipur National Park - Tropical Evergreen Forest

4. Gangotri National Park - Coniferous Forest

Which of the pairs given above are correctly matched?

(a) 1, 2 and 3 only


(b) 1, 2 and 4 only
(c) 1, 3 and 4 only
(d) 2, 3 and 4 only

EXPLANATION:

Bhitarkanika National Park is a large national park in the Indian state of Odisha. It has the
habitat type of mangroves and wetlands.
Blackbuck National Park is a national park in India located at Velavadar in the state of Gujarat.
The habitat of the park region comprises grasslands, shrublands, saline land, and mudflats.
Bandipur National Park is a national park covering 868.63 km2 (335.38 sq mi) in the
chamarajnagar district of the Indian state of Karnataka. The park has a variety of biomes
including dry and moist deciduous forests and shrublands. The wide range of habitats helps
support a diverse range of organisms.
Gangotri National Park is a national park located in the state of Uttarakhand, covering about
2,390 km2 (920 sq mi). Its habitat consists of coniferous forests, alpine meadows, and glaciers.
So, Option (b) is correct.

Q.4) How does ‗Democratic socialism‘ differ from ‗Gandhian Socialism‘?

1. Gandhian Socialism advocated the state to redistribute wealth among people whereas Democratic
Socialism advocated workers' control over factories.

2. Gandhian Socialism rejected the idea of private capital whereas Democratic Socialism accepted the
presence of private capital in the economy.

Select the correct answer using the code given below.

(a) 1 only

(b) 2 only

(c) Both 1 and 2

(d) Neither 1 nor 2

6
EXPLANATION:

According to Gandhi, Socialism does not refer to the nationalization of the means of
production, distribution, and exchange, but faith in God, truth, non-violence, and equality. So,
in simple terms, Gandhian socialism is based on the ideas of non-possession and trusteeship.
Gandhi's ideal is a stateless democracy, in which there is a federation of satyagrahi village
communities, functioning based on voluntary cooperation and dignified and peaceful co-
existence.
According to him, in a stateless democracy, there is decentralization and equality. Non-
profession and bread labour should be the ideal of society. In a stateless democracy, every
individual gets the maximum freedom to devote himself to the service of society according to
his caliber or capacity.
Thus Gandhian Socialism does not advocate the state redistributing wealth among people
rather it advocated a Stateless democracy.
In democratic socialism goals of socialism like equality, and justice is accepted but these goals
are to be attained through democratic means. Democratic socialism allows workers in a factory
to form Trade Unions and Associations. But the law enacted by the democratically elected
government controls the factory and not the workers.
So, Statement 1 is not correct.

According to Gandhi, trusteeship is the only ground on which he can work out an ideal
combination of economics and morals. He believed in the decentralization of work and
Cooperatives and thus he rejected the accumulation of private capital.
In a Democratic Socialism, much of the property is held by the private sectorincluding many
major industries, utilities, and transportation systems and the presence of completely
privatelyowned industries is limited in certain critical areas.
So, Statement 2 is correct.

Q.5) Which of the following are examples of the liberal tendencies in the Constitution of India?

1. Freedom of publication for press

2. Freedom from arbitrary arrest

3. Freedom of Conscience

Select the correct answer using the code given below.

(a) 1 only

(b) 2 only

(c) 1 and 2

(d) 1, 2 and 3

7
EXPLANATION:

The term ‗liberalism‘ is generally associated with

 Demands for greater overall equality


 Defense of individual reason and autonomy
 A tool against moral conservatism
 Cosmopolitanism and humanism
 Free markets

The Constitution of India protects Personal/Individual Liberty through the Fundamental Rights
such as,

 Article 19(a) – Freedom of Speech and Expression which includes Freedom of Press. So,
Statement 1 is correct
 Article 22 – Protection against arrest and detention in certain cases that is no person who
is arrested shall be detained in custody without being informed, as soon as may be, of
the grounds for such arrest nor shall he be denied the right to consult, and to be
defended by, a legal practitioner of his choice.So, Statement 2 is correct
 Article 25 – Right to Freedom of Religion where all persons are entitled to freedom of
conscience. So, Statement 3 is correct

Q.6) Which of the following setup was common for the Maurya, Gupta, and the Pushyabhuti Empire?

1. Their kingdoms extended across the Indo-Gangetic plains including Bengal.


2. They had Pataliputra as their political capital.
3. The western border of these empires extended up to Khandahar.

Which of the statements given above is/are correct?

(a) 1 only
(b) 1 and 2 only
(c) 2 and 3 only
(d) 1, 2 and 3

EXPLANATION:

The Maurya Empire was a geographically extensive Iron age historical power in South Asia based
in Magadha, founded by Chandragupta Empire in 322 BCE, and existed until 185 BCE.The
Maurya Empire was centralized by the conquest of the Indo Gangetic plain.

8
The Gupta Empire was an ancient empire which existed from the early 4th century CE to the late
6th century CE. At its zenith, from approximately 319 to 467 CE, it covered much of the Indian
Subcontinent.

9
The Pushyabhuti dynasty also known as the Vardhana dynasty ruled parts of northern India
during the 6th and 7th centuries. The dynasty reached its zenith under its last ruler Harsha (c.
590–647 CE), whose empire, covered much of north and north-western India.

All of these kingdoms extended along the Indo-Gangetic plains including Bengal. So, Statement
1 is correct.
Both the Mauryan and Guptan empire had their capital at Pataliputra, but the Pushyaputi
empire had its capital at Thaneshwar and Kannauj. So, Statement 2 is not correct.
Both the Mauryan and Guptan empire has their territory extended around the Kandahar region
to its northern border whereas the Pushyabhuti dynasty‘s territory covers only the smaller
portion near the Gangetic plains and it does not extend over the Kandahar region. So,
Statement 3 is not correct.

Q.7) With reference to the colonial rule in India, consider the following statements:

1. The Government of India act of 1919 separated the provincial budgets from the central budget and
authorized the provinces to enact their own budget.
2. The Government of India act of 1919 gave equality in representation to Indians and British in the
Viceroy‘s Executive Council.
3. When direct elections were allowed in 1919, about 10 per cent of the Indian population got voting
rights.

Which of the statement given above is/are correct?

(a) 1 and 2 only


(b) 1 and 3 only
10
(c) 2 and 3 only
(d) 1, 2 and 3

EXPLANATION:

Features of the Government of India Act, 1919:

 It relaxed the central control over the provinces by demarcating and separating the central
and provincial subjects. The central and provincial legislatures were authorised to make
laws on their respective list of subjects. However, the structure of government continued to
be centralised and unitary.

So, Statement 1 is correct.

 It required that the three of the six members of the Viceroy‘s executive Council (other than
the commander-in-chief) were to be Indian.

So, Statement 2 is correct.

Government of India Act of 1935extended the franchise. About 10 per cent of the total
population got the voting right.
So, Statement 3 is not correct.

Q.8) Who among the following is/are associated with the abolition of the practice of Sati in India?

1. Lord William Bentinck


2. Alfonso De Albuquerque
3. Joseph Francois Dupliex

Select the correct answer using the code given below.

(a) 1 only
(b) 1 and 2 only
(c) 2 and 3 only
(d) 1, 2 and 3

EXPLANATION:

Due to the sustained efforts of Raja Ram Mohan Roy, Sati practice was formally banned in
all the lands under Bengal Presidency by Lord William Bentinck on 4 December 1829.So,
Statement 1 is correct.
Afonso de Albuquerque who was the second Portuguese Governor and the real founder of
Portuguese colonial empire in India is known to have abolished the practice of Sati system
in Goa in 1515. So, Statement 2 is correct.
The practice was banned by the Dutch in their colonies in Chinsurah and the French in
Pondicherry.

11
Joseph Marquis Dupleix was Governor-General of French India but was not associated with
the abolition of sati. So, Statement 3 is not correct.

Q.9) Which one of the following statements is not correct?

(a) Light travels at the speed of 3*108 ms-1 in all the medium
(b) Ultraviolet radiation of higher energy is ionizing in nature
(c) Infrared waves produced by hot bodies have lesser energy than visible light
(d) Cosmic rays are highly ionizing in nature

EXPLANATION:

Light travels at the speed of 3*108 m/s only in a vacuum and its speed is not the same in all the
mediums.

For example,

Speed of light in Glass - 2*108 m/s

Speed of light in Diamond – 1.25*108 m/s

Speed of light in Water - 2.25*108 m/s

So, Option (a) is not correct.

Higher-energy UV rays are a form of ionizing radiation. This means they have enough energy to
remove an electron from (ionize) an atom or molecule. Ionizing radiation can damage the DNA
(genes) in cells, which in turn may lead to cancer. But even the highest-energy UV rays don‘t
have enough energy to penetrate deeply into the body, so their main effect is on the skin. So,
Option (b) is correct.

Infrared radiation has longer waves and thus oscillates at a lower frequency and carries less
energy than visible light. So, Option (c) is correct.

Cosmic radiation is a form of ionizing radiation that comes from outer space {from a Star}. A very
small amount of this radiation reaches the earth and they are highly ionizing. So, Option (d) is
correct.

Q.10) With reference to vertical farming, consider the following statements:

1. All vertical farming systems are soilless farming techniques that use air as a medium for plant
growth.
2. The Yield Per Unit Area of Land is much higher in vertical farming than in conventional farming.
3. If non-renewable energy is used to meet these energy demands, vertical farms could produce more
pollution than traditional farms.

Which of the statements given above is/are correct?

12
(a) 1 only
(b) 1 and 2 only
(c) 2 and 3 only
(d) 1, 2 and 3

EXPLANATION:

Through Vertical Farming, food crops can be cultivated easily by planting in vertically stacked
layers in order to save space and use minimal energy and water for irrigation.
It includes soilless farming techniques, among others.
There are three processes usually adopted in the vertical farming systems,
 Hydroponics - method of growing food in water using mineral nutrient solutions without
soil
 Aeroponics – here the roots are suspended in the air and plants grow in a humid
environment without soil and both growing medium and flowing water are absent.
 Aquaponics - combining two words –‗aquaculture‘, which refers to fish farming, and
‗hydroponics‘—the technique of growing plants using water and without soil, to create
symbiotic relationships between the plants and the fish.

Thus the Vertical Farming techniques uses both air and water as a medium for plant growth.
So, Statement 1 is not correct.

Vertical farming systems (VFS) have been proposed as an engineering solution to increase
productivity per unit area of cultivated land by extending crop production into the vertical
dimension.
Vertical farming increases lettuce yield per unit area compared to conventional horizontal
hydroponics. Further increases in yield could be achieved by incorporating artificial lighting in
the VFS.
So, Statement 2 is correct.

Vertical farms also face large energy demands due to the use of supplementary light like LEDs.
Researchers determined that if only solar panels were to be used to meet the energy
consumption of a vertical farm, the area of solar panels required would need to be a factor of

13
twenty times greater than the arable area on a multi-level indoor farm, which will be hard to
accomplish with larger vertical farms.
Example: A hydroponic farm growing lettuce in Arizona would require 15,000 kilojoules (4.2
kWh) of energy per kilogram of lettuce produced. Whereas a traditional outdoor lettuce farm in
Arizona only requires 1100 kJ of energy per kilogram of lettuce grown.So, Statement 3 is
correct.

Q.11) Consider the following statements:

1. In Indian agriculture, the investments made by private far exceed the investments made by the
government.
2. A large majority of Indian agriculturists are small and marginal landowners who at present cannot
use their land as collateral for credit.
3. The majority of credit creation in the Indian agriculture sector is done by Non-Institutional
Lenders.

Which of the statements given above is/are correct?

(a) 2 only
(b) 1 and 2 only
(c) 3 only
(d) 1, 2 and 3

EXPLANATION:

Private Investments in Agriculture usually exceed the Investments made by the Government
in India.
So, Statement 1 is correct.

A goal of agricultural policy in India has been to reduce farmers‘ dependence on informal
credit.
As per the All India Rural Financial Inclusion Survey (NAFIS) report, the average loan

14
taken by agricultural households indicated that 72 per cent of the credit requirement was
met through institutional sources and 28 per cent from non-institutional sources.
So, Statement 3 is not correct

Q.12) Consider the following statements about the Pradhan Mantri Matsya Sampada Yojana:

1. It focuses on the improvement of mariculture and deep-sea fishing only.


2. It aims to replace all the bottom trawlers with deep-sea lines by 2024.
3. It will leverage the use of GEMINI navigation technology to prevent the crossing of maritime borders
by Indian fishermen.

Which of the statements given above is/are not correct?

(a) 1 only
(b) 2 only
(c) 3 only
(d) 1, 2 and 3

EXPLANATION:

The Pradhan Mantri Matsya Sampada Yojana. (PMMSY) is designed to address critical gaps in
fish production and productivity, quality, technology, post-harvest infrastructure and
management, modernisation and strengthening of the value chain, traceability, establishing a
robust fisheries management framework and fishers‘ welfare.
It focuses on the improvement of Mari culture, fishers and fish farmers also.
So, Statement 1 is not correct

The Aims and Objectives of the Pradhan Mantri Matsya Sampada Yojana
(PMMSY) are:
 Harnessing of fisheries potential in a sustainable, responsible,
inclusive and equitable manner.
 Enhancing of fish production and productivity through expansion,
 intensification, diversification and productive utilization of land andwater.
 Modernizing and strengthening of the value chain - post-harvestmanagement and quality
improvement.
 Doubling fishers and fish farmers‘ incomes and generation ofemployment.
 Enhancing contribution to Agriculture GVA and exports.
 Social, physical and economic security for fishers and fish farmers.
 Robust fisheries management and regulatory framework

The Palk Bay scheme was launched in 2017 under the umbrella of Blue Revolution to replace
2,000 bottom trawlers with deep-sea fishing boats. The scheme is financed by the Union
government (50%), and the state Government (20%) with beneficiary participation (30%).
So, Statement 2 is not correct.

15
GEMINI (Gagan Enabled Mariner‘s Instrument for Navigation & Information)was developed by
Hyderabad based Indian National Centre for Ocean Information Services (INCOIS).
For the dissemination of information on disaster warnings, Potential Fishing Zones (PFZ) and
Ocean States Forecasts (OSF) to fishermen, GOI launched the GEMINI device and mobile
application.
It is not connected to PMMSY.
So, Statement 3 is not correct.

Q.13) Consider the following statements:

1. The application of rock phosphate on acidic soils increases the pH levels of the soil but has no
impact on the quantity of phosphorus available to the plants.
2. The application of gypsum on alkaline soil does not affect the pH levels of the soil but improves the
calcium content of the soil.
3. Most of the acidic soils of India are found in the Eastern part of the country while the alkaline soils
are mostly found in the Northwestern part.

Which of the statements given above is/are correct?

(a) 1 only
(b) 1 and 2 only
(c) 2 and 3 only
(d) 2 only

EXPLANATION:

Phosphorus (P) deficiency in agricultural land is one of the main factors which reduce crop
production and yield. To increase crop growth, the availability of P using rock phosphate (RP)
can be enhanced using organic waste with the addition of phosphate solubilizing
microorganisms (PSMs).
Rock phosphate enriched compost with a combined ratio of 50:50 of RP and compost applied
before 7 days of sowing in pot experiments resulted in the maximum nodulation, growth and
productivity of the crops. So, Statement 1 is not correct

Gypsum has been used for centuries to reclaim sodium affected soils (alkali soils). It is effective
since it has good solubility regardless of soil pH. The solubility of gypsum in alkali soils is four
times greater.
Once the gypsum dissociates into calcium and sulphur, the calcium has the greatest attraction
for the soil particle displacing sodium. The calcium will help flocculate (aggregate) the soil
whereas the sodium on the soil causes dispersion. This means that calcium from the gypsum
will improve soil structure and improve plant growth.So, Statement 2 is correct

The alkali soils are largely predominant in the Indo-Gangetic plains encompassing the States of
Punjab, Haryana, Uttar Pradesh & Bihar and partly in states like Chhattisgarh, Rajasthan,

16
Andhra Pradesh, Gujarat, Maharashtra, Karnataka, Andhra Pradesh, Madhya Pradesh and
Tamil Nadu.

About 21 million ha of acid soils are found in the NEH region including Sikkim with the
maximum area under Arunachal Pradesh (6.8 Mha) followed by Assam (4.7 Mha), Meghalaya,
(2.24 Mha), Manipur (2.19 Mha) and Mizoram (2.0 Mha). So, Statement 3 is correct

Q.14) Which of the following properties of the soil cannot be altered by soil management practices?

1. Soil Structure
2. Soil Colour
3. Soil Texture
4. Soil Porosity

Select the correct answer using the code given below:

(a) 1, 2 and 4
(b) 3 only
(c) 2 and 4
(d) 1 and 4

EXPLANATION:

Soil structure is defined by the way individual particles of sand, silt, and clay are assembled.
We can improve the structure of soils by adding organic matter in the form of compost or
composted manure.

Soil colour is influenced by its mineral composition as well as water and organic contents.
Addition of Organic matter tends to make the soil colour darker. During the Humus
formation, the final stage of organic matter breakdown, the colour changes to black.

Soil texture is usually defined as the proportions of sand, silt and clay. We cannot change soil
texture without physically adding or subtracting one of these components.

Soil porosity may be altered by the addition of biochar due to the interaction between biochar
particles and soil aggregates. So, Option (b) is correct.

Q.15) Consider the following statements about ―Biofertilizers‖:

1. These fertilizers have living microorganisms as one of their constituents to accelerate microbial
processes in the soil.
2. Bacillus species improve solubilising many nutrients like phosphorous, silicate and zinc and
Pseudomonas species help in nitrogen fixation and plant growth.

17
3. Fungi are not used as constituents in Biofertilizers.

Which of the statements given above is/are correct?

(a) 1 only
(b) 2 only
(c) 3 only
(d) 1, 2 and 3

EXPLANATION:

Bio-fertilisers are living microorganisms of bacterial, fungal and algal origin. Their mode of
action differs and can be applied alone or in combination.So, Statement 1 is correct and
Statement 3 is not correct.

Pseudomonas and Bacillus species are the predominant plant growth-promoting bacteria.

Many heterotrophic bacteria live in the soil and fix significant levels of nitrogen without
direct interaction with other organisms. Examples of this type of nitrogen-fixing bacteria
include species of Azotobacter, Bacillus, Clostridium, and Klebsiella.

The application of Bacillus-based fertilizers to soil can enhance the plant-available forms of
nutrients in rhizospheres, control disease-causing pathogenic microbial growth and induce
pest defence systems.

Pseudomonas has been reported to have the potential to improve plant biomass, relative
water content, leaf water potential and root adhering soil/root tissue ratio.

The function of zinc is to help the plant produce chlorophyll.So, Statement 2 is not
correct.

Q.16) Consider the following statements regarding Propulsion Systems in India:

1. Glycidyl Azide Polymer and Ammonium Di-Nitramide are green propellants that do not emit
chlorinated exhaust products from rocket motors.
2. Liquid Oxygen and Liquid Hydrogen used in Cryogenic Upper Stage are environment-friendly and
green propellants.
3. It is impossible to manoeuvre a spacecraft in outer space without using fuel and an oxidiser.

Which of the statements given above are correct?

(a) 1 and 3 only


(b) 1 and 2 only
(c) 2 and 3 only
(d) 1, 2 and 3

18
EXPLANATION:

ISRO has made a beginning by developing an eco-friendly solid propellant based on Glycidyl Azide
Polymer (GAP) as fuel and Ammonium Di-Nitramide (ADN) as oxidizer at the laboratory level, which
will eliminate the emission of chlorinated exhaust products from rocket motors and act as green
propellants. So, Statement 1 is correct.

ISRO has already begun the move towards environment-friendly and green propellants with the
acceptance of Liquid Oxygen (LOX)/Liquid Hydrogen (LH2) and LOX/Kerosene-based propulsion
systems for launch vehicles, and the use of electric propulsion for spacecraft. The LOX/LH2 in
combination produces water so it is environment-friendly and it is already being used in the
cryogenic upper stages of GSLV and GSLV Mk-III launch vehicles. So, Statement 2 is correct.

It is possible to manoeuvre a spacecraft in outer space without using fuel and oxidizer while it is in
the coasting period which means the resting time of the flight at the transfer orbit, which helps to
move the spacecraft from one orbit to another. So, Statement 3 is not correct.

Q.17) A tourist travels across a landscape in a certain direction. Along the way, she witnesses the
transformation of vegetation from deciduous forest to short grasses and then to short evergreen shrubs
and bushes. In which direction is the tourist travelling?

(a) West to East in Low Latitude Area


(b) East to West in Mid Latitude Area
(c) North to South at 50 Degrees East Longitude
(d) West to East in High Latitude Area

EXPLANATION:

In low latitude areas, due to the rainfall,it receives from the Trade winds the eastern part of
the continents witnesses the vegetation of Evergreen forest to Deciduous forest to short
grasses in the middle area and desert type on the western side. So, Option (a) is not
correct.

In the mid-latitude area, the eastern side has deciduous forest vegetation under the China
type of climate, the center part has grassland of Steppe type and the western part of the
continent come under the Mediterranean type of climate which has shrubs and bushes due
to hot and dry summer with scanty rainfall. So, Option (b) is correct.

Mostly, north to South at 50 degrees East Longitude covers the area of the desert region,
especially in the middle eastern countries. So, Option (c) is not correct.

In high latitude area, the western part has deciduous forests which comes under the British
type of climate but the middle part has evergreen and coniferous forests and the eastern
part has mixed forests. So, Option (d) is not correct.

19
Q.18) Who among the following is the author of the book ―Indian Women‘s Battle for Freedom‖?

(a) Kamala Devi


(b) Avantibhai Gokhale
(c) Sarojini Naidu
(d) Matangini Hazra

EXPLANATION:

Kamaladevi Chattopadhyay was an Indian social reformer and freedom activist. She was most
remembered for her contribution to the Indian Independence movement; for being the driving
force behind the renaissance of Indian handicrafts, handlooms, and theatre in independent
India; and for the upliftment of the socio-economic standard of Indian women by pioneering the
cooperation. She is the first lady in India to stand in elections from Madras Constituency
although she lost in the elections. she pioneered the path for women in India. The book ―Indian
Women‘s Battle for Freedom‖ is written by her. So, Option (a) is correct.

Q.19) With reference to the recently photographed blackholes, consider the following statements:

1. Sagittarius A* (SgrA*) is a blackhole present at the center of the Milky Way.


2. The black hole photographed at center of the Messier-87 galaxy is much larger than the SgrA*.
3. Blackholes are strong evidence of Einstein‘s General Theory of Relativity.

Which of the statements given above is/are correct?

(a) 1 only
(b) 2 only
(c) 3 only
(d) 1, 2 and 3

EXPLANATION:

A black hole is formed when stars collapse, leading to a space in the universe with an escape
velocity, the speed at which an object must travel to override a planet or an object‘s
gravitational force.

Recently, the Event Horizon Telescope network has captured the second-ever direct image of a
black hole called Sagittarius A* at the center of the Milky Way.Sagittarius A* has four million
times the mass of our Sun and is about 26,000 light-years from Earth. A light-year is the
distance light travels in a year or 9.5 trillion kilometers.

So, Statement 1 is correct.

Using Event Horizon Telescope scientist have found the black hole Sagittarius A* (SgrA*) at
the centre of our Milky way galaxy. In 2019, the same telescope released the first ever image

20
of a black hole M87* – the black hole at the centre of a galaxy Messier 87, which is a
supergiant elliptic galaxy. SgrA* is only one-thousandth the size of M87*.

So, Statement 2 is correct.

Both the Sagittarius and M-87 black hole obeys Einstein‘s theory of general relativity, showing
he was right by a factor of 1,000 in the size scale of a black hole. Relativity has been around for
a century and it is still proving to be accurate. This shows that they are strong evidence of
Einstein‘s General Theory of Relativity. So, Statement 3 is correct.

Q.20) Consider the following statements about ―Compressed Natural Gas‖ (CNG):

1. CNG is lighter than air.


2. CNG has a higher auto-ignition temperature than petrol.
3. CNG is much safer than petrol because of its narrow range of flammability.
4. Petrol has lesser antiknock power than CNG.

Which of the statements given above are correct?

(a) 1 and 2
(b) 1 and 4
(c) 1, 2 and 4
(d) 1, 2, 3 and 4

EXPLANATION:

Compressed natural gas (CNG) is a fuel gas made of petrol which is mainly composed
of methane (CH4), compressed to less than 1% of the volume it occupies at standard
atmospheric pressure. It is lighter than air. Unlike other fuels such as diesel, petrol, or LPG,
which are heavier than air, pools on the ground create a fire hazard and potential pollution
to waterways. Should a CNG leak occur, the gas will disperse rapidly upwards into the
atmosphere and dissipate. So, Statement 1 is correct.

CNG has a higher auto-ignition temperature (540 °C) than petrol and is also much safer than
petrol because of its narrow range (5–15 percent) of flammability, making combustion of CNG
unlikely. So, Statements 2 and 3 are correct.

CNG has a higher octane number and knocking resistance than gasoline and hence CNG-
dedicated engines can have higher compression ratios and therefore higher indicated
efficiencies. So, Statement 4 is correct.

21
Q.21) Which of the following factors will play a role in increasing the value of ICOR in an economy?

1. Improvement in technology

2. Improvement in skill levels of labour

3. Wasteful use of raw materials

4. Increase in time taken for regulatory approvals

Select the correct answer using the code given below.

(a) 1 and 2 only


(b) 1, 2 and 4 only
(c) 2 and 3 only
(d) 3 and 4 only

EXPLANATION:

The incremental capital output ratio (ICOR) is a frequently used tool that explains the
relationship between the level of investment made in the economy and the subsequent
increase in the gross domestic product (GDP). ICOR indicates the additional unit of capital
or investment needed to produce an additional unit of output.

ICOR can be calculated by,

ICOR = Annual Increase in GDP/Annual Investment


A lower ICOR is preferred as it indicates a country's production is more efficient.

Technological improvement and improvement in the skill levels of Labour will increase
Investment and Productivity. So it will lower the value of ICOR making the country more
efficient.

So, Statements 1 and 2 are not correct.

Improper usage of raw materials and delayed process in regulatory approvals will lower the
Investment and Productivity. So it will increase the value of ICOR making the country less
efficient.

So, Statements 3 and 4 are correct.

Q.22) Which of the following measures are likely to help in improving the monetary transmission
mechanism in the Indian banking system?

1. External benchmarking of loans


2. Introduction of Insolvency and Bankruptcy Code
3. Regular conduct of asset quality review by Reserve Bank of India
4. Increase of repo rate

22
Select the correct answer using the code given below.

(a) 2 and 4 only


(b) 1, 2 and 3 only
(c) 1 and 3 only
(d) 1, 2, 3 and 4

EXPLANATION:

Monetary transmission is essentially the process through which the policy action of the central
bank is transmitted to the ultimate objective of stable inflation and growth.

Monetary transmission is the entire process starting from the change in the policy rate by the
central bank to various money market rates such as inter-bank lending rates, to bank deposit
rates, to bank lending rates to households and firms, to government and corporate bond
yields, and to asset prices such as stock prices and house prices, culminating in its impact on
inflation and growth.

 According to RBI, the introduction of the external benchmark system for lending and
deposit rates has helped in improving the monetary transmission by banks. The EBLR
system has also accelerated the pass-through to MCLR-linked loans, as changes in the
benchmark rates lead banks to proactively adjust their deposit rates to protect their NIMs
(net interest margins), thereby improving transmission to overall lending and deposit rates.
So, Statement 1 is correct

The introduction of IBC will result in an easing of the monetary policy stance and improve the
recovery of stressed assets. So, Statement 2 is correct

Being a Comprehensive measure, regular conduct of asset quality review by RBI helps clean
the Balance sheets of the banks and also withstand possible financial shocks. So, Statement
3 is correct

Government should intend to bring down the interest rates on small saving accounts. If Repo
rates are increased, people end up paying higher interest rates for their loans and
disbursement of loans gets decreased. So, Statement 4 is not correct.

Q.23) In the context of the budget of the Government of India, which of the following receipts are non-
debt creating in nature?

1. Cess

2. Profits transferred by RBI

3. Ways and means advances

4. Recovery of past loans

23
Select the correct answer using the code given below.

(a) 3 only
(b) 1, 2 and 4 only
(c) 4 only
(d) 1, 2, 3 and 4

EXPLANATION:

Cess is a form of tax charged/levied over and above the base tax liability of a taxpayer. A cess
is usually imposed additionally when the state or the central government looks to raise funds
for specific purposes. It is non-debt creating in nature. So, Statement 1 is correct.

The Reserve Bank of India (RBI) transfers its Dividends or Surplus money to the Government
of India as a Non-tax revenue for which the government need not pay back the money to the
RBI. So, Statement 2 is correct.

Ways and Means Advances are temporary advances given by the RBI to the government to tide
over any mismatch in receipts and payments. These are debt creating in nature. So,
Statement 3 is not correct.

Recovery of past loans by the government is a liability for the borrower to repay it. It does not
create any debt. So, Statement 4 is correct.

Q.24) When there is a high level of inflation in the economy, which of the following measures could be
counterproductive in controlling inflation?

1. Introducing a universal basic income programme

2. Raising the monetary policy rate

3. Lowering the margin requirements for all types of loans

4. Lowering customs duty on import of raw materials

Select the correct answer using the code given below.


(a) 1, 2 and 3

(b) 2 and 3 only

(c) 2 and 4 only

(d) 1and 3 only

EXPLANATION:

Introducing a Universal basic income (UBI) will increase runway inflation which is a
counterproductive measure. So, Statement 1 is correct.

24
If there is a higher level of Inflation, central banks tighten monetary policy by increasing
interest rates for the inflation rate to get reduced. This will be a productive measure to control
Inflation. So, Statement 2 is not correct.

Margin requirement refers to the difference between the current value of the security offered for
the loan (called collateral) and the value of the loan granted. It is a qualitative method of credit
control adopted by the central bank in order to stabilize the economy from inflation or deflation.
In the case of inflation, the margin requirement is increased so that demand for loans is
decreased. This will be a counter-productive measure to control Inflation. So, Statement 3 is
correct.

Lowering customs duty on the import of raw materials will reduce the Inflation rate which is a
Productive measure. So, Statement 4 is not correct.

Q.25) Which one of the following statements is correct regarding mutual funds?

(a) Mutual funds are risk-free investment options and ensure definite returns for an investor
(b) Mutual funds are extremely volatile instruments but provide the most returns of all investment
options
(c) Mutual funds are subject to market risks, but the risks are minimised through the expertise
of professionals
(d) Mutual funds require investors to invest huge sums of money

EXPLANATION:

Investment risk can be defined as the probability or likelihood of occurrence of losses


relative to the expected return on any particular investment.

Mutual funds are subject to market risk. Although it can't be avoided completely, we can
minimize this risk through expert advice like asset allocation, diversifying the portfolio etc.
So, Option (c) is correct.

Q.26) Consider the following statements:

1. ReCAAP is the first government to government agreement to enhance cooperation against piracy
and armed robbery against ships in Asia.
2. No African countries are contracting parties to ReCAAP but it has parties from Europe and North
America.
3. Both Indian Navy and the Indian Coast Guard conduct anti-piracy patrols in the Indian Ocean.

Which of the statements given above is/are correct?

(a) 1 only

25
(b) 1 and 2 only
(c) 2 only
(d) 1, 2 and 3

EXPLANATION:

The Regional Cooperation Agreement on Combating Piracy and Armed Robbery against Ships in
Asia (ReCAAP) is the first regional government-to-government agreement to promote and enhance
cooperation against piracy and armed robbery against ships in Asia. ReCAAP information-sharing
centre (ISC) was proposed in 1999 as a result of shared concerns specifically related to cases of
piracy and armed robbery, and it came into force in November 2006 after further ratification by
member states. So, Statement 1 is correct.

Till now, 21 States (14 Asian countries, 5 European countries, Australia, and the USA) have
become Contracting Parties to ReCAAP, but no countries are from Africa. So, Statement 2 is
correct.

Generally, both the Indian Navy and Coast Guard will conduct anti-piracy patrols in the Indian
Ocean. But for the ReCAAP agreement, only the Indian Coast Guard is the designated authority to
do patrolling. So, Statement 3 is correct.

Q.27) With reference to the multilateral financing for Asian countries, consider the following statements:

1. Both Public and Private sectors can borrow from New Development Bank (NDB), Asian
Development Bank (ADB), and Asian Infrastructure Investment Bank (AIIB).
2. While NDB can lend to projects in any country, ADB and AIIB can lend to projects only in the
Asian Countries.
3. Grants from Asian Development Fund can be provided by both NDB and ADB.

Which of the statements given above is/are correct?

(a) 1 only
(b) 1 and 2 only
(c) 2 and 3 only
(d) 1, 2 and 3

EXPLANATION:

Asian Development Bank (ADB), established in 1966, is owned by 68 members - 49 are from
within Asia and the Pacific and 19 members outside this region.

The New Development Bank (NDB), was set up by the BRICS countries in 2015 with a more
equitable power structure, to fund infrastructure projects.

AIIB began operations in 2016 with 57 founding Members (37 regional and 20 non-regional). By
the end of 2020, they had 103 approved Members representing approximately 79 percent of the

26
global population and 65 percent of global GDP. Both Public and Private sectors can borrow
from all these three banks. So, Statement 1 is correct.

NDB can mobilize resources for infrastructure and sustainable development projects in BRICS
and other emerging economies, as well as in developing countries.

ADB lend to its member countries which include countries from Asia, Europe and the Pacific
region.

AIIB may provide or facilitate financing to any member, or any agency, instrumentality or
political subdivision thereof, or any entity or enterprise operating in the territory of a member,
as well as to international or regional agencies or entities concerned with the economic
development of Asia.

So, Statement 2 is not correct.

The Asian Development Fund (ADF), offers loans at very low-interest rates and grants to ADB's
poorest borrowing countries, but it can be provided only by the Asian Development Bank and
not by NDB. So, Statement 3 is not correct.

Q.28) With reference to International Energy Agency (IEA), consider the following statements:

1. The IEA primarily aims to improve the energy security of non-OPEC countries by responding to
physical disruptions in the supply of oil and serving as an information source about the
international energy market.
2. The Nuclear Energy Agency is a part of the IEA which focuses on improving the sustainable use of
nuclear energy for peaceful purposes.
3. India is a member of both the International Energy Agency and the Nuclear Energy Agency.

Which of the statements given above is/are correct?

(a) 1 only
(b) 1 and 3 only
(c) 2 and 3 only
(d) 1, 2 and 3

EXPLANATION:

The International Energy Agency is a Paris-based autonomous intergovernmental


organization established in the framework of the Organisation for Economic Cooperation and
Development (OECD) in 1974 in the wake of the 1973 oil crisis to improve the energy security of
non-OPEC countries.The IEA was initially dedicated to responding to physical disruptions in
the supply of oil, as well as serving as an information source on statistics about the

27
international oil market and other energy sectors. It is best known for the publication of its
annual World Energy Outlook. So, Statement 1 is correct.

The Nuclear Energy Agency (NEA) is an intergovernmental agency that is organized under
the Organisation for the Economic Co-operation and Development (OECD) and it is not part of
the IEA. The mission of the NEA is to "assist its member countries in maintaining and further
developing, through international co-operation, the scientific, technological and legal bases
required for the safe, environmentally friendly and economical use of nuclear energy for
peaceful purposes. So, Statement 2 is not correct.

India is not a member country of both the IEA and NEA. But India is an associate country to
the IEA. So, Statement 3 is not correct.

Q.29) As of 2021, India has Free Trade Agreements with which of the following blocs?

1. ASEAN
2. European Union
3. Gulf Cooperation Council
4. Mercosur

Select the correct answer using the code given below.

(a) 1, 2 and 3
(b) 1 only
(c) 2 and 4
(d) 3 and 4

EXPLANATION:

 The ASEAN–India Free Trade Area (AIFTA) is a free trade area among the ten member states
of the Association of Southeast Asian Nations (ASEAN) and India. The initial framework
agreement was signed on 8 October 2003 in Bali and the final agreement was on 13 August
2009.The free trade area came into effect on 1 January 2010.
 India and the European Union are set to hold an initial round of negotiations on a Free Trade
Agreement to conclude an FTA by late 2023 or early 2024
 India is inthe process of beginning deeper engagements with the Gulf Cooperation Council
(GCC) countries as early as to finalize a free trade agreement (FTA) with the group of nations,
but it is still in talks.
 Only a Preferential Trade Agreement between India and MERCOSUR was signed in New Delhi
on January 25, 2004, and five annexes to this Agreement were signed and incorporated on
March 19, 2005, but as of now, Free Trade Agreement was not signed between them.

So, Option (b) is correct.

28
Q.30) Which of the following organization recently released the ―Guidelines for the prevention and
suppression of the smuggling of Wildlife on ships engaged in International Maritime Traffic‖ to provide
recommendations for Government and Private Agencies regarding the criminal activity of Wildlife
trafficking?

(a) International Organisation of Airports and Seaports Police


(b) World Wildlife Fund for Nature
(c) International Chambers of Shipping
(d) International Maritime Organisation

EXPLANATION:

 International Maritime Organization (IMO) adopts new guidelines called ―Guidelines for the
prevention and suppression of the smuggling of Wildlife on ships engaged in International
Maritime Traffic‖ to combat wildlife smuggling and the Global shipping to focus on bringing
down the illegal networks exploiting maritime supply chains to traffic wildlife.
 This endorsement by the United Nations specialized agency sends a strong message on the
growing international engagement against the illegal wildlife trade (IWT) and its impacts on
global biodiversity, directly threatening the survival of many species in the wild.

So, Option (d) is correct.

Q.31) With reference to the social reformists in Modern India, consider the following statements:

1. Swami Virajananda rechristened Mool Shankar as Dayanand Sarawati with the task of spreading
Vedic knowledge.
2. Swami Dayanand Saraswati made Karma Siddhanta, Rebirth, Brahmacharya, and Sanyasa as the
four pillars of his philosophy.
3. Swami Dayanand Saraswati was the first to give the call for Swaraj as "India for Indian" in 1876.

Which of the statements given above is/are correct?

(a) 1 only
(b) 2 only
(c) 1 and 2
(d) 1, 2 and 3

EXPLANATION:

Swami Virajananda entrusted Mool Shankar with the task of spreading Vedic knowledge
throughout the society and rechristening him as Rishi Dayanand Saraswati. Maharishi Dayanand
was a believer in Hinduism just as the Vedas have outlined, devoid of any corruption and
embellishments.

So, Statement 1 is correct.

29
Dayananda advocated the doctrine of Karma and Reincarnation. He emphasized the Vedic ideals
of brahmacharya, including celibacy and devotion to God.So, Statement 2 is correct.

Maharshi Dayanand Saraswati was an Indian philosopher, social leader and founder of the Arya
Samaj, a reform movement of Vedic dharma. He was the first to give the call for Swaraj as "India
for Indian" in 1876.So, Statement 3 is correct.

Q.32) Why did Mahatma Gandhi never return to Sabarmati Ashram, where he started the salt march
(Dandi March), in the then Bombay Province?

(a) Due to the failure of abandoning the Civil Disobedience Movement due to the lack of support from
the masses
(b) He had decided not to return to Sabarmati till independence for India was attained
(c) He was imprisoned immediately after the launch of the salt march which failed to achieve its
objective
(d) Due to the failure of the Round Table conference, he decided not to return to the Sabarmati
Ashram

EXPLANATION:

When Gandhi started his Padayatra (foot march) in 1930 from Sabarmati Ashram to Dandi for
the Salt Satyagraha, he had decided not to return to Sabarmati till independence for India was
attained. So, Option (b) is correct.

Q.33) With reference to the tribal uprisings in Colonial India, consider the following statements regarding
―Birsa Munda‖:

1. He created a faith called ‗Birsait‘ as a challenge to the religious conversion activities of the British
Raj and their oppression of the tribal communities.
2. He joined the Sardar movement for the land and rights of the tribals in 1894.
3. The states of Chhattisgarh and Jharkhand were created on his birth anniversary in 2000 in
recognition of his contribution to the national freedom movement.
4. He spearheaded the demonstration against the Britishers with his most famous tribal movement —
the Munda Rebellion.

Which of the statements given above are correct?

(a) 1, 2 and 3 only


(b) 1, 2 and 4 only
(c) 1, 3 and 4 only
(d) 2, 3 and 4 only

30
EXPLANATION:

Having gained awareness of the British colonial ruler and the efforts of the missionaries to
convert tribals to Christianity, Birsa started the faith of ‗Birsait‘. Soon members of the
Munda and Oraon community started joining the Birsait sect and it turned into a challenge
to British conversion activities. So, Statement 1 is correct.

Wanting to fight the Britishers, Birsa joined the Sardar movement for the land and rights of
the tribals in 1894. So, Statement 2 is correct.

The new states of Chhattisgarh, Uttarakhand, and Jharkhand were formed on the 1st, 9th,
and 15th of November 2000, respectively.

In recognition of his impact on the national movement, the state of Jharkhand was created
on his birth anniversary in 2000.

Chhattisgarh was formed from Madhya Pradesh for Socio-Political purposes. So, Statement
3 is not correct.

He spearheaded a tribal religious millenarian movement – ‗Munda Rebellion‘ that arose in the
Bengal Presidency (now Jharkhand) in the late 19th century, during the British Raj, thereby
making him an important figure in the history of the Indian independence movement.So,
Statement 4 is correct.

Q.34) The places such as ―Serampore‖ and ―Tranquebar‖ in 17th Century India are known for which one
of the following?

(a) These were the places where the British East Indian Company set up a printing press for the first
time
(b) These were the settlements of the Danish East Indian Company
(c) These were the major trade centres in the western coastal region of India
(d) These were the places where Portugal set up their factories in India for the first time

EXPLANATION:

The Danish East India Company was established in 1616, they founded a factory at Tranquebar
near Tanjore, on the eastern coast of India. Their principal settlement was at Serampore near
Calcutta. The Danish factories, which were not important at any time, were sold to the British
government in 1845. The Danes are better known for their missionary activities than for
commerce.Eg, Serampore Missionary.

So, Option (b) is correct.

31
Q.35) Consider the following statements:

1. The Interim Government was officiated immediately after the Shimla Plan in 1945.

2. BR Ambedkar was the first law minister in the interim Government in 1945.

3. The Interim Government functioned under the presidency of the British Viceroy.

Which of the statements given above is/are correct?

(a) 2 only

(b) 3 only

(c) 2 and 3

(d) 1, 2 and 3

EXPLANATION:

The interim Government of India was formed on September 2, 1946, to oversee the transition
of the country from a British colony to an independent republic.

The Shimla Conference in 1945 was convened in order to agree on the Wavell Plan for Indian
self-government, which provided for separate representations on communal lines. Both the
plan and the conference failed on account of the Muslim League and the Indian National
Congress not agreeing.

So, Statement 1 is not correct.

Jogendra Nath Mandal was the first law minister in the interim Government in 1945.So,
Statement 2 is not correct.

The executive branch of the interim government was served by the viceroy's executive council,
headed by the Viceroy of India.

President of the Executive Council (Viceroy and Governor-General of India): Viscount Wavell
(till February 1947); Lord Mountbatten (from February 1947). So, Statement 3 is correct.

Q.36) With reference to the Indian States along the international boundary of India, consider the
following statements:

1. This state is bounded by two neighbouring countries.

2. It is the only state in India to have a river border on both East and West with the two neighbouring
countries bordering it.

3. Within India, this state shares borders with 3 neighbouring states.

Which one of the following states is described by the statements given above?

(a) Jammu and Kashmir

32
(b) Uttar Pradesh

(c) Assam

(d) Mizoram

EXPLANATION:

Jammu Kashmir is a union territory of India that is bounded to the east by the Indian union
territory of Ladakh, to the south by the Indian states of Himachal Pradesh and Punjab, and to
the southwest by Pakistan. So, Option (a) is not correct.

Uttar Pradesh is bordered by the state of Uttarakhand and the country of Nepal to the north, the
state of Bihar to the east, the states of Jharkhand and Chhattisgarh to the southeast, the state
of Madhya Pradesh to the south, and the states of Rajasthan and Haryana and the national
capital territory of Delhi to the west. Thus it is bounded by one neighbouring country and seven
states. So, Option (b) is not correct.

Assam borders two neighbouring countries such as Bhutan and Bangladesh. But it borders
seven neighbouring states such as Meghalaya, Arunachal Pradesh, Nagaland, Manipur,
Mizoram, Tripura, and West Bengal. So, Option (c) is not correct.

Mizoram borders two neighbouring countries such as Myanmar and Bangladesh and three
bordering states such as Tripura, Assam, and Manipur. The Tiau river borders the state and
Myanmar on the eastern side and the river Karnapuli borders the state and Bangladesh onthe
western side. So, Option (d) is correct.

Q.37) Consider the following pairs of waterfalls and their source rivers in India.

Waterfalls Source rivers

1. Nohkalikai Falls in Meghalaya - Barak River

2. Hogenakkal falls in Tamilnadu - Cauvery River

3. Dudhsagar Falls in Goa - Mandovi River

Which of the pairs given above is/are correctly matched?

(a) 1 only

(b) 1 and 2 only

(c) 2 and 3 only

(d) 1, 2 and 3

33
EXPLANATION:

The Nohkalikai waterfall is located in the Indian state of Meghalaya, near Cherrapunji, the
second wettest place on Earth. It is mainly fed by the rainwater collected at the top of a small
plateau located around the town of Sohra.

The Barak River flows through the states of Meghalaya, Manipur,Mizoram,Assam, Tripura, and
Nagaland in India.So, Pair 1 is not correct.

Hogenakkal falls in Tamil Nadu and is fed by the Cauvery River. So, Pair 2 is correct.

Dudhsagar Falls (―Sea of Milk‖) in Goa is fed by the Mandovi River. So, Pair 3 is correct.

Q.38) If a foreign tourist wants to explore the mountain railway system, recognised by UNESCO, then the
tourist must visit which of the following places?

1. Darjeeling in West Bengal

2. Nilgiri in Tamil Nadu

3. Shimla in Himachal Pradesh

4. Jammu in Jammu and Kashmir

5. Matheran in Maharashtra

Select the correct answer using the code given below.

(a) 1, 2 and 3 only

(b) 1, 2, 4 and 5 only

(c) 1, 2 and 4 only

(d) 1, 2, 3, 4 and 5

EXPLANATION:

The Mountain Railways of India which was accorded the UNESCO World Heritage status are,

 The Darjeeling Himalayan Railway, West Bengal (Northeast India) in 1999


 The Nilgiri Mountain Railway, Tamil Nadu (South India) in 2005
 The Kalka Shimla Railway, Himachal Pradesh (Northwest India) in 2008
 Chhatrapati Shivaji Terminus, Maharashtra in 2004

So, Option (a) is correct

The Central Railways has proposed the Matheran toy train in Maharashtra (Neral-Matheran
Light Railway) for the 2021 edition of the UNESCO-Greece Melina Mercury International
Prize.

34
The UNESCO-Greece Melina Mercury International Prize for Safeguarding and Management
of Cultural Landscapes established in 1995 is an award to outstanding examples of actions
to safeguard and enhance the world‘s cultural landscapes, a category of World Heritage.

The Matheran Hill Railway and Kangra Valley Railwayare on the tentative list of UNESCO
World Heritage Sites.

Q.39) With reference to the Mahakali River, consider the following statements:

1. The Mahakali River forms the international boundary between India and Nepal and flows into River
Ghaghra.
2. The entire catchment area of the Mahakali River lies in India.
3. The Pancheshwar Multipurpose Project, developed in the Mahakali River, is jointly built by India
and Russia for benefit of Nepal.

Which of the statements given above is/are correct?

(a) 1 only
(b) 1 and 2 only
(c) 2 and 3 only
(d) 1, 2 and 3

EXPLANATION:

The Mahakali river also called Sarda River/ Kali river originates at Kalapani and forms the
border between India and the western border of Nepal for a considerable distance.

It joins the Ghaghra River, a tributary of the Ganges. So, Statement 1 is correct.

35
The catchment area of Mahakali is around 15,260 sq km, a large part of which (9,943 sq
km) lies in Uttarakhand and the rest lies in Nepal. So, Statement 2 is not correct.

The Pancheshwar Dam, a joint venture of India and Nepal for irrigation and hydroelectric
power generation was proposed on the Mahakali river at Dharchula in Uttarakhand. Russia
plays no role in the construction of this project. So, Statement 3 is not correct.

Q.40) Consider the following worms:

1. Green Worm
2. African Nightcrawler
3. Tiger Worm

Which of the above organisms are capable of producing ―Vermicompost‖?

(a) 1 only
(b) 1 and 2 only
(c) 2 and 3 only
(d) 1, 2 and 3

EXPLANATION:

Generally, the earthworm is added with other essential materials for producing
vermicompost. The following worms are also good for vermicomposting.

 Red Wiggler
 European Nightcrawler
 Indian or Malaysian Blue
 African Nightcrawler
 Tiger Worm

Green worms are not capable of vermicomposting. So, Option (c) is correct.

Q.41) In India, which of the following crops are mostly grown in the soil that is formed after weathering of
Basalt rocks?

(a) Cotton
(b) Rice
(c) Rubber
(d) Tapioca

36
EXPLANATION:

Basalt rocks are the rocks that are formed due to the rapid cooling of lava which has a high
content of minerals like Magnesium and Iron. It contains less silica content.

Black soil is formed by the weathering or breaking of igneous (basaltic) rocks and also by the
cooling or solidification of lava from the volcano eruption.

These soils are also known as the ‗Regur Soil‘. The soil is deficient in nitrogen, phosphate,
and organic matter but rich in potash, calcium, and magnesium. The colour of the soil
ranges from deep black to grey.

In India, they cover the plateaus of Maharashtra, Saurashtra, Malwa, Madhya Pradesh, and
Chhattisgarh and extend in the southeast direction along the Godavari and the Krishna
valleys. Black soil is ideal for growing cotton and is also known as black cotton soil. So,
Option (a) is correct.

Q.42) Which one of the following statements is correct about the Red Sea?

(a) It is located in between Africa and Asia


(b) It is located in between Iran and Saudi Arabia
(c) For Pakistan, it is very important to develop its Gwadar port
(d) To access all the central Asian republics, the Red Sea provides the shortest route from the Arabian
sea

37
EXPLANATION:

The Red Sea is located between Asia and Africa. So, Option (a) is correct.

The Persian Gulf is located between Iran and Saudi Arabia. So, Option (b) is not correct.

38
Gwadar Port Authority is situated on the Arabian Sea at Gwadar in Balochistan province
of Pakistan and is under the administrative control of the Maritime Secretary of Pakistan
and operational control of the China Overseas Port Holding Company.

So, Option (c) is not correct.

The Central Asian region consistsof Kazakhstan, Kyrgyzstan, Tajikistan, Turkmenistan,


and Uzbekistan.

The Red Sea does not connect the Arabian sea with Central Asian countries rather it
connects with European Countries viathe Mediterranean Sea. So, Option (d) is not
correct.

Q.43) Consider the following statements about the ―Zero Budget Natural Farming‖ (ZBNF):

1. As per the Indian Council of Agricultural Research, ZBNF would result in a tremendous increase in
the production of crops.
2. The Integrated Production System will focus primarily on improving soil health than on crop
productivity.
3. ZBNF reduces the cost of production in agriculture nearly to zero as it only uses the natural
resources available in and around the crops.

Which of the statements given above is/are correct?

(a) 1 only
(b) 1 and 2 only
(c) 2 and 3 only
(d) 1, 2 and 3

EXPLANATION:

Recently, an expert committee set up by the Indian Council of Agricultural Research (ICAR)
stated that large scale adoption of Zero Budget Natural Farming (ZBNF) would result in a

39
'tremendous reduction' in the production of agricultural crops.So, Statement 1 is not
correct.

It refers to the use of homemadeamendments from readily available ingredients. These


inputs are intended to promote soil health, close nutrient cycling loops, and provide greater
water retention in soil rather than increasing crop productivity. So, Statement 2 is correct.

‗ZeroBudget‘ refers to lower use of purchased inputs, and reduced involvement of


agribusiness,reducing debt incurred by farmers. So, Statement 3 is correct.

Q.44) Which one of the following is the most salt-tolerant crop?

(a) Sugar cane


(b) Grams
(c) Sesame
(d) Sugar beet

EXPLANATION:

Sugar beet is considered to be a salt-tolerant crop. Sugar beet can improve its salt tolerance
through the following regulation strategies: regulation of gene expression, accumulation of
osmotic-adjustment substances, vacuolar salt sequestration, regulation of polyamine
synthesis, and antioxidant regulation.

So, Option (d) is correct.

Q.45) A symbiotic relationship between Fungus and Plants is found in

(a) Lichen
(b) Mycorrhiza
(c) Pneumatophore
(d) Bacteriophage

EXPLANATION:

Lichens are commonly recognized as a symbiotic association of a fungus and chlorophyll-


containing partner, either green algae or cyanobacteria, or both. The fungus provides a
suitable habitat for the partner by protecting and anchoring the body and the algae provide
photosynthetically fixed carbon as an energy source for the fungi.
So, Option (a) is not correct.

Mycorrhizae are symbiotic(mutualistic) relationships that form between fungi and plants. The
fungi colonize the root system of a host plant, providing increased water and nutrient
absorption capabilities by putting out mycelia while the plant provides the fungus with
carbohydrates formed from photosynthesis which makes easy access to food for the fungus.
So, Option (b) is correct.

40
Pneumatophores are specialized root structures that grow out from the water surface and
facilitate the aeration necessary for root respiration in hydrophytic trees such as many
mangrove species. So, Option (c) is not correct.

Bacteriophages are viruses that host bacteria but are harmless to humans. To reproduce,
they get into a bacterium, where they multiply, and finally, they break the bacterial cell open
to release the new viruses. Therefore, bacteriophages kill bacteria. So, Option (d) is not
correct.

Q.46) Consider the following statements about the Agriculture Infrastructure Fund:

1. The cost of credit is capped for 7 years for all eligible beneficiaries in the agriculture sector.
2. The targeted credit creation through the fund is 1 lakh crore rupees in 10 years.
3. Financial assistance through the fund will be made available for agriculture infrastructure projects
only and not for farming activities.

Which of the statements given above is/are correct?

(a) 1 only
(b) 1 and 2
(c) 2 only
(d) 1, 2 and 3

EXPLANATION:

Agriculture Infrastructure Fund shall provide a medium - long-term debt financing facility for
investment in viable projects for post-harvest management Infrastructure and community
farming assets through interest subvention and financial support.

All loans under this financing facility will have an interest subvention of 3% per annum up to a
limit of Rs.2 crores. This subvention will be available for a maximum period of 7 years.

So, Statement 1 is correct.

The duration of the scheme is from FY2020 to FY2029 (10 years). Under the scheme, Rs.1
Lakh Crore will be provided by banks and financial institutions as loans with interest
subvention.

So, Statement 2 is correct.

A financing facility will be provided for funding Agriculture Infrastructure Projects at farm-gate
& aggregation points (Primary Agricultural Cooperative Societies, Farmers Producer
Organizations, Agriculture entrepreneurs, Start-ups, etc.) and not for farming activities. So,
Statement 3 is correct.

41
Q.47) Which of the following are the characteristics of Visible Light Communication (VLC)?

1. No electromagnetic interference
2. Penetration through walls
3. Higher frequency and bandwidth than radio waves
4. Low power consumption

Select the correct answer using the code given below.

(a) 1 and 2 only


(b) 2 and 3 only
(c) 2 and 4 only
(d) 1, 3 and 4 only

EXPLANATION:

Visible light communication (VLC) is a data communications variant that uses visible
light between 400 and 800 THz (780–375 nm). VLC is a subset of optical wireless
communications technologies. The characteristics of VLC are as follows,

 Immune from electromagnetic interference


 It cannot penetrate through walls and it is used in indoor environments/short range
 It has high bandwidth and frequency when compared to radio waves
 Low powered consumption
 Non-licensed channels

So, Option (d) is correct.

Q.48) Which of the following are the possible undesirable effects that can be caused due to satellite-based
internet services like Starlink?

1. Kessler Syndrome
2. Havana Syndrome
3. Interference in optical astronomy
4. Interference in radio astronomy

Select the correct answer using the code given below.

(a) 1 and 2 only


(b) 2 and 3 only
(c) 2 and 4 only
(d) 1, 3 and 4 only

42
EXPLANATION:

As of early January 2022, SpaceX had launched more than 1,900 Starlink satellites overall.
These satellites had formed a Constellation in the space providing broadband service in select
areas around the world, as part of a beta-test program with download speeds of between 100
Mb/s and 200 Mb/s and latency as low as 20 milliseconds.

The Kessler Syndrome is a phenomenon in which the amount of junk in orbit around Earth
reaches a point where it just creates more and more space debris, causing big problems for
satellites, astronauts, and mission planners.

Almost 2,000 active satellites whiz around our planet, along with nearly 3,000 dead satellites
and 34,000 pieces of ―space junk‖ larger than 10 centimeters in size have occupied Space. As
Earth‘s orbit becomes jam-packed with satellites, satellite-based internet services like
Starlink could lead to the Kessler Syndrome in the worst-case scenario. So, Statement 1 is
correct

Havana Syndrome is a set of unexplained medical symptoms first experienced by U.S. State
Department personnel stationed in Cuba beginning in late 2016.Those affected report a range
of conditions including dizziness, headache, fatigue, nausea, anxiety, cognitive difficulties,
and memory loss of varying severity.

So, Statement 2 is not correct

The American Astronomical Society (ASS) in a report stated that these Mega satellite
constellations may cause, Light Pollution in astronomy. The report said the sky may brighten
by a factor of two to three due to the diffuse reflection of sunlight off the spacecraft.

Satellites are already an issue for astronomers studying celestial objects in deep space. The
lights from the satellite constellations of the Starlink project may interfere with optical
astronomy by disrupting the astronomers while taking long-exposure shots of the sky with
their telescopes.

So, Statement 3 is correct

Often, astronomers will study radio waves coming from distant objects to learn more about
them, especially hot bodies like stars that emit super intense X-rays that can be measured
from Earth. Some astronomers are concerned that the radio frequencies these satellite
constellations will be transmitting on will also interfere with radio observations of the
Universe. So, Statement 4 is correct

Q.49) Consider the following passage:

―This method aims at extracting secrets from a chip or a system, through measurement and analysis of
physical parameters such as supply current, execution time, and electromagnetic emission‖

Which one of the following is explained in the passage given above?


43
(a) Side Channel Attack
(b) DDoS Attack
(c) Spear-phishing attack
(d) XSS attack

EXPLANATION:

Side-channel attacks (SCAs) aim at extracting secrets from a chip or a system, through
measurement and analysis of physical parameters. Examples of such parameters include
supply current, execution time, and electromagnetic emission.

So, Option (a) is correct.

A side-channel attack uses an oscilloscope, a type of electronic test device. It measures the
power consumption and then compares its behavior to random PIN codes. Analyzing the
measurements of each PIN digit helps build a database that can then be used with a script
to guess the digits one by one.

Side-channel attacks can even prove effective against air-gapped systems that have been
physically segregated from other computers or networks. Additionally, they may also be used
against virtual machines (VMs) and in cloud computing environments where an attacker and
target share the same physical hardware.

These attacks pose a serious threat to modules that integrate cryptographic systems, as
many side-channel analysis techniques have proven successful in breaking an
algorithmically robust cryptographic operation (for example, encryption) and extracting the
secret key.

A distributed denial-of-service (DDoS) attack is a malicious attempt to disrupt the normal


traffic of a targeted server, service, or network by overwhelming the target or its surrounding
infrastructure with a flood of Internet traffic.

DDoS attacks achieve effectiveness by utilizing multiple compromised computer systems as


sources of attack traffic.

So, Option (b) is not correct.

Spear-phishing is a targeted attempt to steal sensitive information such as account


credentials or financial information from a specific victim, often for malicious reasons.

It is a potent variant of phishing, a malicious tactic that uses emails, social media, instant
messaging, and other platforms to get users to divulge personal information or perform
actions that cause network compromise, data loss, or financial loss.

So, Option (c) is not correct.

44
Cross-site scripting (also known as XSS) is a web security vulnerability that allows an
attacker to compromise the interactions that users have with a vulnerable application.XSS
attacks occur when an attacker uses a web application to send malicious code, generally in
the form of a browser side script, to a different end-user.

So, Option (d) is not correct.

Q.50) Which one of the following statements is not correct?

(a) The strong nuclear force is charge dependent


(b) Gravitational force is the weakest of all fundamental forces
(c) Electromagnetism is a fundamental force
(d) The weak interaction is responsible for radioactive decay

EXPLANATION:

The strong nuclear force binds protons and neutrons in a nucleus.The strong nuclear

force is the strongest of all fundamental forces, about 100 times the electromagnetic force in
strength.

It is charge-independent and acts equally between a proton and a proton, a neutron and a
neutron, and a proton and a neutron.

So, Option (a) is not correct.

The gravitational force is the force of mutual attraction between any two objects by their
masses. It is a universal force. It is the weakest of all fundamental forces.

So, Option (b) is correct.

The Four Fundamental Forces of Nature are

 Gravitational force
 Weak Nuclear force
 Electromagnetic forceSo, Option (c) is correct.
 Strong Nuclear force

While the other three fundamental forces hold things together, the weak interaction also
called weak nuclear force plays a greater role in things falling apart, or decaying. The weak
force is stronger than gravity, but it is only effective at very short distances. It governs the
radioactive decay of unstable subatomic particles such as mesons and initiates the nuclear
fusion reaction that fuels the Sun. So, Option (d) is correct.

45
Q.51) Which of the following countries are members of the ―Five Eyes‖ intelligence-sharing alliance?

1. China
2. Germany
3. USA
4. Canada
5. India
6. Australia

Select the correct answer using the code given below:

(a) 2, 3, 5 and 6
(b) 1, 2, 4, and 5
(c) 1, 2, 3, 4, and 5
(d) 3 and 4

EXPLANATION:

The Five Eyes alliance is an intelligence-sharing arrangement between five English-speaking


democracies. It evolved during the Cold War as a mechanism for monitoring the Soviet Union
and sharing classified intelligence. It is often described as the world's most successful
intelligence alliance.

India is not a member of this alliance.

So, Option (d) is correct.

Q.52) With reference to International Arbitration, consider the following statements:

1. Only those States that are parties to the Convention on Pacific Settlement of disputes can be sued
against in the Permanent Court of Arbitration.
2. The award of the Permanent Court of Arbitration can be appealed or set aside in other
international arbitral centres that administers based on UNCITRAL‘s model law.
3. India is not a party to the Convention on Pacific Settlement of disputes but is a member of
UNCITRAL.

Which of the statements given above is/are correct?

(a) 1 only
(b) 1 and 2 only
(c) 2 and 3 only
(d) 1, 2 and 3

46
EXPLANATION:

The Permanent Court of Arbitration (PCA) is a non-UN international organization located in


the Netherlands. Unlike a judicial court in the traditional sense, the PCA provides the
services of an arbitral tribunal to resolve disputes that arise out of international agreements
between member states, international organizations, or private parties. Parties to the
convention on the pacific settlement of disputes of 1899and 1907 are automatically parties
to the PCA. Hence only those States that are parties to the Convention on Pacific Settlement
of disputes can be sued against in the Permanent Court of Arbitration. So, Statement 1 is
correct.

In the Vodafone tax case, PCA ruled in Vodafone‘s favour. Now, India has the option of filing
an appeal against the decision of the Hague-based Permanent Court of Arbitration (PCA) at
the Singapore-based appellate tribunal.
All these international arbitration courts follow UNCITRAL‘s Model Law

So, Statement 2 is correct.

India is a party to the Convention on Pacific Settlement of disputes signed in 1899 and it is
also a member of UNCITRAL. So, Statement 3 is not correct.

Q.53) Which of the following international agencies has the mandate of fighting terrorism among their
other functions?

1. Interpol
2. UN Peacekeeping Force
3. ATMIS
4. Europol

Select the correct answer using the code given below.

(a) 2 and 3
(b) 2, 3 and 4
(c) 1 and 4
(d) 1, 2, 3 and 4

EXPLANATION:

Interpol is an international organization that facilitates worldwide police cooperation and crime
control. Headquartered in Lyon, France, it is the world's largest international police organization,
with seven regional bureaus worldwide. It provides investigative support, expertise, and training
to law enforcement worldwide, focusing on three major areas of transnational
crime, terrorism, cybercrime, and organized crime.

47
Peacekeeping by the United Nations was founded in 1945 and it acts as an instrument developed
by the organization as a way to help countries torn by conflict to create the conditions for lasting
peace and to fight against terrorism.

The African Union Transition Mission in Somalia (ATMIS) is an active African Union transition
and drawdown mission from peacekeeping operations in Somalia to fight against terrorism.

Europol is the law enforcement agency of the European Union (EU) formed in 1998 to
handle criminal intelligence and combat serious international organized crime and terrorism
through cooperation between competent authorities of EU member states. So, Option (d) is
correct.

Q.54) Which one of the following best defines the ―P vs NP problem‖, seen in the news recently?

(a) The risk associated with a genetic mutation in GM crops


(b) Unsolved problem in computer science
(c) Unknown nature of Dark Energy and Dark Matter
(d) Limitations in IPv4 compared to IPv6

EXPLANATION:

Polynomial versus Nondeterministic polynomial problem (P vs NP), in computational


complexity (a subfield of theoretical computer science and mathematics), is the question of
whether all so-called NP problems are P problems.

P problem can be solved in ―polynomial time,‖ which means that an algorithm exists for its
solution such that the number of steps in the algorithm is bounded by a polynomial function
of n, where n corresponds to the length of the input for the problem. Thus, P problems are said
to be easy, or tractable.

A problem is called NP if its solution can be guessed and verified in polynomial time, and
nondeterministic means that no particular rule is followed to make the guess.

P problems are all NP by definition. For if we can solve a problem in polynomial time, then we
can also verify its solution in polynomial time – by solving it.But we don‘t know if all NP
problems are P. So the great debate in mathematics and computer science circles is, is P the
same as NP goes on.

So, Option (b) is correct.

Q.55) Consider the following statements about the International Cricket Council (ICC):

1. ICC is an intergovernmental organization for governing all international and domestic cricket.
2. All domestic cricket trophies and leagues must follow the rules of the game as provided by the ICC.
48
3. The headquarters of ICC is in the Lord‘s Cricket Ground in London.

Which of the statements given above is/are not correct?

(a) 1 and 2 only


(b) 1 and 3 only
(c) 2 and 3 only
(d) 1, 2 and 3

EXPLANATION:

The International Cricket Council (ICC) is the world governing body of cricket and not all
domestic cricket will be overseen by it. Few domestic leagues are following the rules of their
respective country's governing body. So, Statement 1 is not correct.

There is no need that all domestic cricket trophies and leagues must adhere to the rules of ICC
but it may adhere to their guidelines. For example, Ranchi Trophy in India is based on the BCCI
rules {BCCI largely follows the guidelines of ICC with few modifications}. So, Statement 2 is not
correct.

The headquarters of ICC is presently in Dubai since 2005 but it was in the Lord‘s Cricket
Ground in London from 1909 to 2005. So, Statement 3 is not correct.

Q.56) Consider the following statements regarding the Prime Minister Wi-Fi Access Network Interface,
recently seen in the news:

1. It is an ambitious program of the Department of Telecommunications.


2. Under this, Public Data Offices will be established on the lines of Public Call Offices.
3. It is an attempt by the government to elevate wireless internet connectivity only in the Railway
stations.
4. It aims to connect all silo Wi-Fi networks for ease of use and proliferate broadband usage for the
masses.

Which of the statements given above are correct?

(a) 1, 2 and 4 only


(b) 1, 2 and 3 only
(c) 1 and 3 only
(d) 3 and 4 only

EXPLANATION:

PM-Wifi Access Network Interface is an ambitious program of the Department of Telecom,


Government of India to connect all silo Wi-Fi networks for ease of use and proliferate broadband
usage for the masses.So, Statements 1 and 4 are correct.

49
Public Data Offices (PDOs) will be established on the lines of Public Call Offices (PCOs). These
PDOs will establish, operate, and maintain only the WANI-compliant Wi-Fi access points to
deliver broadband services to their subscribers.These PDOs will either provide internet on their
own or will lease from some other Internet Service Provider (ISP). PDO Aggregators (PDOAs) will
also be set up. These will perform the function of authorization and accounting of the numerous
PDOs. So, Statement 2 is correct.

It is being seen as an attempt by the government to elevate wireless internet connectivity


throughout the country. So, Statement 3 is not correct.

Q.57) Consider the following statements about the ―Periodic Labour Force Survey‖ (PLFS) in India:

1. It is compiled by the Labour Bureau under the Union Ministry of Labour and Employment.
2. It aims to provide the Worker Population Ratio, Labour Force Participation Rate and
Unemployment Rate in the short time interval of three months.
3. It aims to estimate employment and unemployment indicators in both rural and urban areas
annually.

Which of the statements given above is/are correct?

(a) 1 only
(b) 1 and 2 only
(c) 2 and 3 only
(d) 1, 2 and 3

EXPLANATION:

National Statistical Office (NSO) {Ministry of Statistics }launched the Periodic Labour Force Survey
(PLFS) in April 2017 and not the Labour ministry.

The objective of PLFS is primarily two-fold:

 To estimate the key employment and unemployment indicators (viz. Worker Population Ratio,
Labour Force Participation Rate, Unemployment Rate) in the short time interval of three months
for the urban areas only in the ‗Current Weekly Status (CWS).

 To estimate employment and unemployment indicators in both ‗Usual Status‘ (ps+ss) and CWS
in both rural and urban areas annually.

So, Option (c) is correct.

Q.58) Consider the following statements about the ―Global Network Against Food Crises‖:

1. It is an international alliance of the United Nations, the European Union, governmental and non-
governmental agencies.
50
2. It is an alliance of humanitarian and development actors united by the commitment to tackle the
root causes of food crises and promote a sustainable solution.
3. People in Ethiopia, southern Madagascar, South Sudan and Yemen were classified in the most
severe phase of acute food insecurity.

Which of the statements given above is/are correct?

(a) 1 only
(b) 3 only
(c) 1 and 2 only
(d) 1, 2 and 3

EXPLANATION:

Global Network Against Food Crises is an international alliance of the United Nations, the
European Union, and governmental and non-governmental agencies working to tackle food
crises together. The report focuses on those countries and territories where the magnitude and
severity of the food crisis exceed the local resources and capacities. In these situations, the
mobilization of the international community is necessary. So, Statement 1 is correct.

It is an alliance of humanitarian and development actors united by the commitment to tackle the
root causes of food crises and promote sustainable solutions through shared analysis and
knowledge, strengthened coordination in evidence-based responses, and collective efforts across
the Humanitarian, Development, and Peace (HDP) nexus. So, Statement 2 is correct.

It reveals that around 193 million people in 53 countries or territories experienced acute food
insecurity at crisis or worse levels (IPC/CH Phase 3-5) in 2021. This represents an increase of
nearly 40 million people compared with the already record numbers of 2020. Of these, over half
a million people (570 000) in Ethiopia, southern Madagascar, South Sudan, and Yemen were
classified in the most severe phase of acute food insecurity Catastrophe (IPC/CH Phase 5) and
required urgent action to avert a widespread collapse of livelihoods, starvation, and death. So,
Statement 3 is correct.

Q.59) The Natural Capital Finance Alliance

1. was launched at the UN Conference on Sustainable Development or Rio+ 20 Earth Summit in


2012.
2. was launched by the Global Environment Facility along with the Worldwide Fund for Nature.
3. is a worldwide finance led initiative to integrate natural capital considerations into financial
products and services.

Select the correct answer using the code given below.

(a) 1 and 2

51
(b) 2 only
(c) 2 and 3
(d) 1 and 3

EXPLANATION:

The Natural Capital Finance Alliance and its Secretariat were originally formed to support the
signatories of the Natural Capital Declaration (NCD), which was launched at the Rio+20 conference
in 2012. The declaration has been signed by the CEOs of more than 40 financial institutions from
around the world. It formalizes their commitment to the integration of natural capital
considerations into financial sector reporting. So, Statements 1 and 3 are correct.

It was launched by UNEP Finance Initiative and the UK-based non-governmental organization,
Global Canopy Programme (GCP). So, Statement 2 is not correct.

Q.60) Consider the following statements about the ―Card Tokenisation Services‖:

1. It applies to every device that connects to the Internet.


2. The token can be used for contactless card transactions at the POS terminal and QR code
payments.
3. As per RBI guidelines, no entity in the payment chain other than the Card Issuer can store the
actual card data.

Which of the statements given above is/are correct?

(a) 1 only
(b) 1 and 2 only
(c) 3 only
(d) 1, 2 and 3

EXPLANATION:

Card Tokenisation services refer to the replacement of actual card details with an alternate
code called the ―token‖, which shall be unique for a combination of card, token requestor (i.e.
the entity which accepts a request from the customer for tokenization of a card and passes it
on to the card network to issue a corresponding token) and device (referred hereafter as
―identified device‖). It applies to every device that connects to the Internet. So, Statement 1 is
correct.

Tokenisation has been allowed through mobile phones and/or tablets for all use
cases/channels (e.g., contactless card transactions, payments through QR codes, apps, etc.
So, Statement 2 is correct.

52
a. With effect from January 1, 2022, no entity in the card transaction/payment chain, other
than the card issuers and/or card networks, shall store the actual card data. Any such data
stored previously shall be purged as per the RBI guidelines. So, Statement 3 is correct.

Q.61) Consider the following statements:

1. The per capita net national income of India at constant prices has never seen a decline since 1951.
2. The decline in per capita net national income at constant prices need not necessarily cause a
decline in net national income at constant prices for a particular year.
3. India witnessed a decline in net national income at constant prices for the first time in 2020-21.

Which of the statements given above is/are correct?

(a) 1 and 2
(b) 2 only
(c) 1 and 3
(d) 3 only

EXPLANATION:

The per capita net national income of India at constant prices has seen a decline in the past.
For example, in 1957-58, 1965 to 1967, 1972-73, etc. So, Statement 1 is not correct.

Per Capita Income = (Net National Income) / (Population)

The value of per capita income not only depends on the value of net national income, it also
depends on the population of that particular country. Let‘s take an example where per capita
income of a country with a population of 100 is 1000 then its net national income is NNI =
1000*100=100000

In another example where per capita income of a country with a population of 120 is 900 then
its net national income is

NNI = 900*120= 108000

The decline in per capita net national income at constant prices need not necessarily result in
the decline of Net National income it may increase too due to the population of that country.
So, Statement 2 is correct.

India witnessed a decline in net national income at constant prices for the first time in 1957-
1958 and not in 2020-21. So, Statement 3 is not correct.

53
Q.62) With reference to the religions of India, consider the following statements:

1. Both the sculptures of Reclining Buddha and Ananta-Shayana Vishnu depict them to be attaining
the stage of greatest salvation.
2. The Avadhana Shataka have stories connected with Buddha and Mauryan King Ashoka, the
Nirvana Shataka has stories connected only to Buddha and his Disciples.
3. Buddha was included as an avatar of Lord Vishnu during the reign of Sungas in the Post-Mauryan
Period.

Which of the statements given above is/are not correct?

(a) 1 only
(b) 1 and 2
(c) 2 only
(d) 1, 2 and 3

EXPLANATION:

A reclining Buddha statue or image represents The Buddha during his last illness (not the
stage of greatest salvation) is a stage before entering Parinirvanawhichis the stage of great
salvation after death that can only be attained by enlightened souls. The Buddha‘s death
came when he was 80 years old, in a state of meditation, in Kushinagar in eastern Uttar
Pradesh, close to the state‘s border with Bihar.

Anantashayana Vishnu is a sculpture of sleeping Lord Vishnu on serpent Ananta. It is a


symbolic representation of the cosmic balance of finity within infinity.So, Statement 1 is
not correct.

54
The Avadanasataka (A Hundred Glorious Deeds) is an anthology of one hundred biographical
stories of Buddhist legends in Sanskrit. It contains two chapters one is called jataka stories
(though they are not called such in the text). Stories 11-20 (the second chapter of the text)
tell of Śākyamuni Buddha's past-life encounters with buddhas of the past. Stories 31-40
take place at times of no past buddhas and demonstrate in large part the extraordinary
deeds of Śākyamuni-to-be.

Nirvana Shatakam also known as Atma Shatakam is a prakarana grantha consisting of six
verses by Adi Shankaracharya.The instruction emphatically conveyed by the six verses of
Nirvana Shatakam is that identification with the body, mind, and senses is the root cause of
all sorrow and that it should be given up and one should realize one‘s real nature as none
other than the supreme Brahman. This realization is what is known as Liberation.

So, Statement 2 is not correct.

Puranas mention Buddha as the 9th avatar of Lord Vishnu and not the Shungas. This is
believed only by the Hindus and not by the Buddhists.

So, Statement 3 is not correct.

Q.63) Consider the following statements about the Kanheri caves:

1. There are more than 100 Buddhist caves.


2. These are deeply nestled within the Sanjay Gandhi National Park in Maharashtra.
3. These are part of the Ajanta Buddhist caves in Maharashtra.

Which of the statements given above are correct?

(a) 1 and 2 only


(b) 2 and 3 only
(c) 1 and 3 only
(d) 1, 2 and 3

55
EXPLANATION:

Kanheri, which has over 100 caves, comes under the purview Archaeological Survey of India (ASI).
The caves contain Buddhist sculptures and relief carvings, paintings and inscriptions dating from
1st century CE up to 10th century CE.

So, Statement 1 is correct.

It is located within the forests of Sanjay Gandhi National Park at Borivali in Mumbai,
Maharashtra, they are a collection of rock-cut monuments.These caves have artistic evidence of all
the three vehicles of Buddhism: Hinayana, Mahayana, and Vajrayana. The earliest reference to
Kanherl is ascribed to Fa-Hein. It flourished under the patronage of Satavahana, Traikutakas,
Vakatakas and Silaharas.

So, Statement 2 is correct.

These are not the part of Ajanta caves because the Kanheri caves are located on the Mumbai side
whereas the Ajanta caves are located in the Aurangabad district of Maharastra. So, Statement 3
is not correct.

Q.64) Consider the following statements:

1. There is a huge Nandi bull made from a single granite stone.


2. The temple is built in the Vijayanagar architectural style and has beautiful sculptures adorning the
walls.
3. It has many murals paintings from the Vijayanagar Era and the famous sculpture of the snake on
the Nagalinga.

Which one of the following temples is described by the statements given above?

(a) Brihadeeshwara Temple, Thanjavur, Tamilnadu


(b) Konark Sun Temple, Puri, Odisha
(c) Lepakshi Temple, Anantapur, Andhra Pradesh
(d) Ambika Mata Temple, Udaipur, Rajasthan

EXPLANATION:

The village of Lepakshi is located in the District of Anantapur, about 13 km from Hindupur. The
village is famous for its three shrines, which are dedicated to Lord Vishnu, Lord Shiva, and Lord
Veerabhadra.

The Veerabhadra Temple was constructed by two brothers, Viranna and Virupanna. The temple
is built in the Vijayanagar architectural style and has beautiful sculptures adorning the walls.

There is a huge Nandi bull made from a single granite stone.

56
Lepakshi has many murals from the Vijayanagar Era and the famous sculpture of the snake on
the Nagalinga.

Local legend claims that the 'naga' was carved from a single block of stone.

So, Option (c) is correct.

Q.65) Consider the following statements about the Lord Macaulay‘s ‗Minute on Indian Education‘:

1. It acknowledged the historic role of Sanskrit and Arabic literature in the subcontinent.
2. It recommended English as the medium of instruction for higher studies and vernaculars at the
school level.

Which of the statements given above is/are correct?

(a) 1 only
(b) 2 only
(c) Both 1 and 2
(d) Neither 1 nor 2

EXPLANATION:

Macaulay‘s Minute (1935),held the view that ―Indian learning was inferior to European
learning‖—which was true as far as physical and social sciences in the contemporary stage were
concerned. It also acknowledged the historic role of Sanskrit and Arabic literature in the
subcontinent. So, Statement 1 is correct.

In 1854, Charles Wood prepared a despatch on an educational system for India. It was
considered the ―Magna Carta of English Education in India‖, and it was the first comprehensive
plan for the spread of education in India. It recommended English as the medium of instruction
for higher studies and vernaculars at the school level.So, Statement 2 is not correct.

Q.66) Consider the following statements:

1. Supreme Court established under the Regulating Act of 1773 was competent to try all British
subjects and Indians all over British India.
2. Lord Warren Hastings established District Diwani Adalats to try both civil and criminal disputes.
3. Lord Cornwallis established the gradation civil courts for both Hindu and Muslim laws.

Which of the statements given above is/are correct?

(a) 1 only
(b) 2 only
(c) 3 only
(d) 1, 2 and 3

57
EXPLANATION:

Regulating Act of 1773 paved the way for the establishment of the Supreme Court of
Judicature at Calcutta with full power &authority to hear and determine all complaints about
any crimes and also to entertain, hear and determine any suits or actions against any of His
Majesty‘s subjects in Bengal, Bihar, and Orissa.So, Statement 1 is not correct.

The Supreme Courts at Madras and Bombay were established by King George – III on 26
December 1800 and on 8 December 1823 respectively.

The India High Courts Act 1861 was enacted to create High Courts for various provinces and
abolished Supreme Courts at Calcutta, Madras, and Bombay and also the Sadar Adalats in
Presidency towns. These High Courts had the distinction of being the highest Courts for all
cases till the creation of the Federal Court of India under the Government of India Act 1935.

Warren Hasting established two courts for resolving disputes

 District Diwani Adalat – for civil disputes


 District Fauzdari Adalats – for criminal disputes
So, Statement 2 is not correct.

Lord Cornwallis is known for the establishment of the sovereignty of law in India because he
established the gradation civil courts for both Hindus and Muslims such as Munsiff Court,
Registrar Court, District Court, Sadar Diwani Adalat, and King-in-Council.So, Statement 3 is
correct.

Q.67) Consider the following statements about the ―Licensing Regulations of 1823‖:

1. These were directed chiefly against Indian language newspapers and newspapers which are edited
by Indians.
2. According to these regulations, starting or using a press without a licence was a penal offence.
3. These regulations were enacted during the Governor-Generalship of John Adams.

Which of the statements given above is/are correct?

(a) 2 only
(b) 1 and 3 only
(c) 3 only
(d) 1, 2 and 3

EXPLANATION:

According to these regulations, starting or using a press without a license was a penal
offense. Later on, the Act was extended to cover journals, pamphlets, and books. These

58
restrictions were directed chiefly against Indian language newspapers or those edited by
Indians.So, Statements 1 and 2 are correct.

The acting Governor-General, John Adams who had a reactionary view enacted the Licensing
Regulations of 1823. So, Statement 3 is correct.

Q.68) With reference to the Anglo-Maratha Wars, consider the following statements:

1. Under the treaty of Purandhar, Raghunathrao ceded the territories of Salsette and Bassein to the
English.
2. The Treaty of Wadgaon forced the Bombay government to relinquish all territories acquired by the
English since 1775.
3. Under the treaty of Bassein, the Peshwa agreed to surrender the city of Surat.

Which of the statements given above is/are correct?

(a) 2 only
(b) 2 and 3 only
(c) 1 and 3 only
(d) 1, 2 and 3

EXPLANATION:

Raghunathrao, unwilling to give up his position in power, sought help from the English at
Bombay and signed the Treaty of Surat in 1775. Under that treaty, Raghunathrao ceded the
territories of Salsette and Bassein to the English along with a portion of the revenues from the
Surat and Bharuch districts.

So, Statement 1 is not correct.

During the First Anglo Maratha war, the English army was surrounded on all sides by the
Marathas and cut off from food and water supplies. The English surrendered by mid-January
1779 and signed the Treaty of Wadgaon that forced the Bombay government to relinquish all
territories acquired by the English since 1775.

So, Statement 2 is correct.

During the Second Anglo Maratha war, under the treaty of Bassein (1802), the Peshwa agreed to

 To cede to the Company territories yielding an income of Rs 26 lakh


 To surrender the city of Surat
 To give up all claims for Chauth on the Nizam‘s dominions
 To accept the Company‘s arbitration in all differences between him and the Nizam or the
Gaekwad, etc

So, Statement 3 is correct.

59
Q.69) Which of the following provisions were introduced in the Constitution by Constitutional
Amendment Acts?

1. Limiting the size of the Legislative Council in the states to 1/3rd of the size of the Legislative
Assembly.

2. Establishment of a common High Court for two or more states.

3. The words ―unity and integrity of the nation‖ in the Preamble.

Select the correct answer using the code given below.

(a) 1, 2 and 3

(b) 2 and 3

(c) 1 and 2

(d) 2 only

EXPLANATION:

The Seventh Constitutional Amendment Act of 1956 amended two articles Article 170 and 171.

Under clause (1) of Article 171, the maximum strength of the Legislative Council of a State is
fixed at one-fourth of the strength of the Legislative Assembly of that State. Although in the
larger States, like Uttar Pradesh and Bihar, this maximum is adequate, it leads to difficulties in
the case of the smaller States. It is, therefore, proposed to alter the maximum to one-third of the
strength of the Legislative Assembly.

So, Statement 1 is correct.

The Constitution of India provides for a high court for each state, the Seventh Constitutional
Amendment Act of 1956 amended Articles 230, 231, and 232 stating,

Article 231 - Establishment of a common High Court for two or more States. Parliament may by
law establish a common High Court for two or more States or two or more States and a Union
territory. So, Statement 2 is correct.

The 42nd Constitutional Amendment Act of 1976 amended the Preamble to the constitution as

(a) for the words "SOVEREIGN DEMOCRATIC REPUBLIC" and the words
"SOVEREIGNSOCIALIST SECULAR DEMOCRATIC REPUBLIC" shall be substituted; and

(b) for the words "unity of the Nation", the words "unity and integrity of the Nation" shall be
substituted. So, Statement 3 is correct.

Q.70) Consider the following statements:

1. A devoutly religious person can be secular in outlook.

60
2. A secular state would completely withdraw from all private spheres of a citizen‘s life.

3. Secularism as a political philosophy came into existence in the Indian Constitution since 1976.

Which of the statements given above is/are not correct?

(a) 1 and 2

(b) 2 and 3

(c) 3 only

(d) 1, 2 and 3

EXPLANATION:

It is upto the person who is devoutly religious to be either religious or secular in outlook.
Mahatma Gandhi as a devoutly religious person is completely secular in outlook to promote
unity among the citizens of India.

So Statement 1 is correct.

India as a Secular nation did not completely withdraw from all Private Spheres of a Citizen‘s
life. It promotes all religions in India through its Fundamental Rights Articles 14,15 and 16 by
not discriminating against anyone based on religion and treating every religion equally. Article
25 guarantees the religious rights of individuals and article 26 protects collective freedom of
religion but these are subject to public order, morality, and health but not subject to other
provisions relating to the Fundamental Rights. Though India is a secular nation it interferes in
the private sphere of a citizen's life but a positive sphere.

So Statement 2 is not correct.

Only the word Secular was added to the Constitution through the 42 nd Constitutional
Amendment Act in 1976. But Secularism as a Political Philosophy was in existence since the
enactment of the Constitution. Example Article 14, 15, and 16 denote the Secularistic
philosophy of the Indian Constitution.

So, Statement 3 is not correct.

Q.71) Which of the following is/are the function(s) of the ―Election Commission of India‖?

1. Preparing Electoral Roll for elections to Legislative Councils in the states.

2. Conducting elections to the office of the speaker of Lok Sabha.

3. Assigning symbols for political parties for local body elections.

Select the correct answer using the code given below.

(a) 1 only

61
(b) 1 and 2

(c) 2 only

(d) 1, 2 and 3

EXPLANATION:

The superintendence, direction, and control of the preparation of the electoral rolls for, and
the conduct of, all elections to Parliament and the Legislature (includes both Assembly and
Council) of every State and of elections to the offices of President and Vice President held
under this Constitution shall be vested in the Election Commission of India.

So, Statement 1 is correct.

Newly elected Members of Parliament from the Lok Sabha elect the Speaker among
themselves. Usually, a member belonging to the ruling party is elected Speaker.The Speaker
(along with the Deputy Speaker) is elected from among the Lok Sabha members by a simple
majority of members present and voting in the House.

Thus the election for the office of the speaker of Lok Sabha is not conducted by the Election
commission of India and by the members of Lok Sabha on the date fixed by President
presided over by the Protem Speaker.

So, Statement 2 is not correct.

The superintendence, direction, and control of the preparation of electoral rolls and the
conduct of all elections to the panchayats including assigning symbols for political parties for
the local body shall be vested in the state election commission.

So, Statement 3 is not correct.

Q.72) With reference to the borrowings by the Centre and the States under the Constitution of India,
consider the following statements:

1. Borrowing limits on the states can be imposed by the Parliament.

2. States can only borrow money with the consent of the Centre.

3. States can borrow money from foreign sources.

Which of the statements given above is/are correct?

(a) 1 only

(b) 2 only

(c) 3 only

(d) None of the above

62
EXPLANATION:

According to Article 293,

The executive power of a State extends to borrowing within the territory of India upon the
security of the Consolidated Fund of the State within such limits if any, as may from time to
time be fixed by the Legislature of such State by law and to the giving of guarantees within such
limits, if any, as may be so fixed.

The Centre through Finance Ministry can impose borrowing limits on the state if there is still
outstanding any part of a loan that has been made to the State by the Government of India or
by its predecessor Government, or in respect of which guarantee has been given by the
Government of India or predecessor Government.

Thus the Parliament has no role to play in the borrowing limits of the state and only the
Government can impose limits that too in certain cases only.

So, Statement 1 is not correct.

Normally, States can borrow money within the territory of India upon the security of the
Consolidated Fund of the State through its Executive action. So, Statement 2 is not correct.

A State may not without the consent of the Government of India raise any loan if there is still
outstanding any part of a loan which has been made to the State by the Government of India or
by its predecessor Government, or in respect of which guarantee has been given by the
Government of India or its predecessor Government.only borrow money with the consent of the
Centre.

The Union Cabinet in 2017 has approved the financially sound State Government entities to
directly borrow frombilateral ODA (Official Development Assistance) partners and external
bilateral funding agencies for implementation of vital infrastructure projects, but subject to
fulfillment of certain conditions and all repayments of loans and interests to the funding
agencies will be directly remitted by the concerned borrower. The concerned State Government
will furnish a guarantee for the Loan. The Government of India will provide a counter-guarantee
for the loan.

Thus States can borrow money from foreign sources. So, Statement 3 is correct.

Q.73) Consider the following statements about the Supreme Court of India:

1. The money and fees received by the Supreme Court of India are not a part of the Consolidated
Fund of India.

2. The pensions payable to the officers of the Supreme Court shall be charged on the Consolidated
Fund of India.

63
Which of the statements given above is/are correct?

(a) 1 only

(b) 2 only

(c) Both 1 and 2

(d) Neither 1 nor 2

EXPLANATION:

According to Article 146(3),

The administrative expenses of the Supreme Court, including all salaries, allowances, and
pensions payable to or in respect of the officers and servants of the Court, shall be charged upon
the Consolidated Fund of India, and any fees or other money taken by the Court shall form part
of that Fund.

So, Statement 1 is not correct and Statement 2 is correct.

Q.74) With reference to the procedures of Lok Sabha, consider the following statements:

1. A member of a Parliamentary Committee can resign by submitting a letter addressed to the


Chairman of that Committee.

2. The Deputy Speaker of Lok Sabha automatically becomes the chairperson of that committee to
which she/he is appointed.

Which of the statements given above is/are correct?

(a) 1 only

(b) 2 only

(c) Both 1 and 2

(d) Neither 1 nor 2

EXPLANATION:

A member of a Parliamentary Committee can resign by writing under their own hand
addressed to the Speaker/Chairman of that House. So, Statement 1 is not correct.

The Deputy Speaker has one special privilege, that is, whenever he is appointed as a member
of a parliamentary committee, he automatically becomes its chairman.So, Statement 2 is
correct.

64
Q.75) Consider the following statements:

1. Soil mulching requires keeping bare soil covered with straw, leaves, and the like, even when the
land is in use.
2. Zero tillage is the process where the crop seed will be sown through drillers without prior land
preparation and disturbing the soil where previous crop stubbles are present.

Which of the statements given above is/are correct?

(a) 1 only
(b) 2 only
(c) Both 1 and 2
(d) Neither 1 nor 2

EXPLANATION:

Soil mulching is the application of a layer of material (mulch) to the surface of an area of
soil.It requires keeping all bare soil covered with straw, leaves, and the like, even when the
land is in use.As mulch prevents rain from hitting the soil directly, the impact of the water
drops is reduced.Erosion is curtailed, moisture retained, and beneficial organisms, such as
earthworms, are kept in place. Mulching adds organic matter to the soil.Mulch decreases
water loss due to evaporation.The same set of benefits is also offered by not tilling the soil. So,
Statement 1 is correct.

Tillage is an agricultural land preparation through mechanical agitation which includes


digging, stirring, and overturning.Zero tillage is the process where the crop seed will be sown
through drillers without prior land preparation and disturbing the soil where previous crop
stubbles are present. Zero tillage not only reduces the cost of cultivation it also reduces the
soil erosion, crop duration, irrigation requirement, and weed effect which is better than
tillage. Zero tillage (ZT) is also called No-Tillage or Nil Tillage. So, Statement 2 is correct.

Q.76) During the winter season, when we see the lakes, we find that the upper layer is frozen. However,
the water that is present below the frozen surface remains liquid. Which one of the following is the
reason for this phenomenon?

(a) Ice is denser than water


(b) Specific Heat Capacity of water
(c) Ice is a bad conductor of heat
(d) Anomalous expansion of water

EXPLANATION:

When water is heated from 0 degrees Celsius to 4 degrees Celsius it contracts rather than
expands. Water begins to expand once it reaches 4 degrees Celsius. This behaviour of water

65
between 0 and 4 degrees Celsius is known as anomalous expansion of water.

When it is cold outside, the top layer of a body of water cools first. The top layer‘s
temperature drops to 4°C. The top layer then becomes denser than the lower layers and
descends to the bottom of the body of water. This process is repeated indefinitely to produce
a temperature gradient. The temperature in the lowest layer remains constant at 4°C.

The coldest layer is the topmost layer. Eventually, the top layer freezes, forming an insulating
blanket that, to some extent, prevents further freezing. If there had been no anomalous
expansion, the water body would have completely frozen. So, Option (d) is correct.

Q.77) Which of the following innovation(s) is/are indigenously developed in India?

1. AmbiTAG
2. Ubreathe life
3. NIRAMAI

Select the correct answer using the code given below.

(a) 1 only
(b) 2 only
(c) 3 only
(d) 1, 2 and 3

Indian Institute of Technology in Punjab‘s Ropar has developed a first-of-its-kind Internet of


Things (IoT) device - AmbiTag.AmbiTag is a USB-shaped device that continuously records
the temperature of its immediate surroundings from -40℃ to 80℃ in any time zone for a full
90 days on a single charge. The device records real-time ambient temperature during the
transportation of perishable products, body organs, blood, vaccines, etc.So, Statement 1 is
correct

IIT Ropar‘s startup company introduces World‘s first ‗Plant based‘ smart air-purifier
―Ubreathe Life‖ which amplifies the air purification process in indoor spaces. These indoor
spaces can either be hospitals, schools, offices, and Homes. The technology uses living,
breathing plants for the filtration of contaminants. The novel technology used in this
product is the ‗Urban Munnar Effect‘ along with patent-pending ―Breathing Roots‖ to
exponentially amplify the phytoremediation process of the plants. Phytoremediation is a
process by which plants effectively remove pollutants from the air.So, Statement 2 is
correct

Niramai ("Non-Invasive Risk Assessment with Machine Intelligence") is a novel software-


based medical device that detects early-stage breast cancer simply and privately in women of
all age groups and breast densities. The solution developed is a low-cost, accurate,

66
automated, portable, contactless, radiation-free, and painless cancer screening tool with no
known side effects. It is developed by a Bengaluru-based start-up NIRAMAI Health Analytix.
So, Statement 3 is correct

Q.78) Which of the following is nota Fundamental Right guaranteed in the Constitution of India?

1. Wearing dress other than that prescribed through a Law in Government Schools.
2. A Member of Parliament questioning a Minister in the Parliament about a bilateral relation, which
could potentially strain India‘s relation with that particular country.

Select the correct answer using the code given below.

(a) 1 only
(b) 2 only
(c) Both 1 and 2
(d) Neither 1 nor 2

EXPLANATION:

In a recent controversial Hijab issue, the High court of Karnataka has declared that the
prescription of uniforms in schools does not violate the right to freedom of speech and
expression or the right to privacy of the students guaranteed in the Constitution of India.

The court also said, ―There is absolutely no scope for a complaint of manifest arbitrariness or
discrimination as per Articles 14 and 15 of the Constitution, when the dress code is equally
applicable to all the students, regardless of religion, language, gender, or the like.‖

Thus Wearing a dress other than that prescribed through a Law in Government Schoolsis not
a Fundamental Right.

So, Statement 1 is not correct.

Asking questions is an inherent and unfettered parliamentary right of members of parliament.


It is during the Question Hour that the members can ask questions on every aspect of
administration and Governmental activity. Government policies in national as well as
international spheres come into sharp focus as the members try to elicit pertinent information
during the Question Hour.

Thus it is not a Fundamental right but it is a Parliamentary Privileges given to the Members of
Parliament under Article 105.

So, Statement 2 is not correct.

Q.79) Which of the following is antithetical to Article 50 of the Indian Constitution?

1. Involvement of Executive in selection and appointment of Judges of Supreme Court


67
2. Chief Justice of India giving assent to the bills passed by the parliament when the office of
President and Vice President falls vacant at the same time.
3. Creating a tribunal with equal numbers of Judicial and Non-Judicial members.

Select the correct answer using the code given below.

(a) 1 only
(b) 1 and 3 only
(c) 2 and 3 only
(d) 1, 2 and 3

EXPLANATION:

Article 50 states that,

Separation of judiciary from the executive. The State shall take steps to separate the judiciary
from the executive in the public services of the State.

Thus the involvement of the Executive in the selection and appointment of Judges of the
Supreme Court violates the supreme feature of the Indian Constitution of Independence of
Judiciary and is also antithetical to Article 50. So, Statement 1 is correct.

According to THE PRESIDENT (DISCHARGE OF FUNCTIONS) ACT, 1969,

In the event of the occurrence of vacancies in the offices of both the President and the Vice-
President, by reason in each case of death, resignation or removal, or otherwise, the Chief
Justice of India or, in his absence, the seniormost Judge of the Supreme Court of India
available shall discharge the functions of the President until a new President elected following
the provisions of the Constitution to fill the vacancy in the office of the President enters upon
his office or a new Vice-President so elected begins to act as President under article 65 of the
Constitution, whichever is earlier.

Tribunal is a quasi-judicial body that is neither a court nor an executive body. Article 50 talks
only about the Executive and Judiciary and not about Legislative or quasi-judicial bodies. So,
Statements 2 and 3 are not correct.

Q.80) ―Financial Inclusion Index‖ is released by

(a) World Bank

(b) International Monetary Fund

(c) Reserve Bank of India

(d) NITI Aayog

68
EXPLANATION:

The Financial Inclusion Index is released by the Reserve Bank of India to gauge how many
people in the country have access to banking and financial services and products and the
usage and quality of such facilities.

It is based on the three dimensions of financial inclusion, viz., 'Access', 'Usage' and 'Quality'
with weights as 35, 45, and 20 percent respectively.

The index captures information on various aspects of financial inclusion in a single value
ranging between 0 and 100, where 0 represents complete financial exclusion and 100 indicates
full financial inclusion.So, Option (c) is correct.

Q.81) The national master plan ―Prime Minister Gati Shakti‖ incorporates which of the following
segments of our economy?

1. Inland waterways
2. Defence corridors
3. Textile clusters
4. Agriculture zones
5. Industrial corridors

Select the correct answer using the code given below.

(a) 1, 3 and 4 only


(b) 2, 3 and 5 only
(c) 3 and 4 only
(d) 1, 2, 3, 4 and 5

EXPLANATION:

 Gati Shakti is National Master Plan for Multi-modal Connectivity (a digital platform) that will
bring 16 Ministries including Railways and Roadways together for integrated planning and
coordinated implementation of infrastructure connectivity projects.
 It will incorporate the infrastructure schemes of various Ministries and State Governments like
Bharatmala, Sagarmala, inland waterways, dry/land ports, UDAN, etc.
 Economic Zones like textile clusters, pharmaceutical clusters, defence corridors, electronic
parks, industrial corridors, fishing clusters, and Agri zones will be covered to improve
connectivity & make Indian businesses more competitive.
 It will also leverage technology extensively including spatial planning tools with ISRO imagery
developed by BiSAG-N (Bhaskaracharya National Institute for Space Applications and
Geoinformatics).

So, Option (d) is correct.

69
Q.82) In the Indian context, consider the following statements:

1. The oilseed production has steadily increased since 2016-2017 after showing a fluctuating trend
before that.
2. The domestic oilseed production in India has exceeded its consumption levels at present.
3. India is the second largest consumer and number one importer of vegetable oil.

Which of the statements given above is/are correct?

(a) 1 only
(b) 1 and 3 only
(c) 3 only
(d) 1,2 and 3

EXPLANATION:

India is one of the major oilseeds growing countries. The oilseed production in India has steadily
increased from 2016-to 17 onward after showing a fluctuating trend before that. The oilseed
production in India has grown by almost 43 percent from 2015-16 to 2020-21. The oil
production in India has however lagged behind its consumption necessitating the import of
edible oils. So, Statement 1 is correct and Statement 2 is not correct.

India is the world‘s second largest consumer and number one importer of vegetable oil. As
urbanization increases in developing countries, dietary habits and traditional meal patterns are
expected to shift towards processed foods that have a high content of vegetable oil. Vegetable oil
consumption in India is, therefore, expected to remain high due to high population growth and
consequent urbanization. So, Statement 3 is correct.

Q.83) Consider the following statements regarding the Heatwaves:

1. It generally occurs over plains of the northwestern part of India, Central, East and North
Peninsular India.
2. It is occurring only in May, which is the peak month of the heat wave in India.
3. It is occurring only in the inter-continental regions in India and not in the coastal states due to
oceanic influence.

Which of the statements given above is/are correct?

(a) 1 only
(b) 1 and 2 only
(c) 2 and 3 only
(d) 1, 2 and 3

70
EXPLANATION:

Heatwaves generally occur over plains of northwest India, Central, East, and north Peninsular
India. Maximum temperatures of more than 45°C were observed mainly over Rajasthan and
Vidarbha region in May causing a heatwave. So, Statement 1 is correct.

In India Heat waves typically occur from March to June, and in some rare cases, even extend
till July.The peak month of the heatwave over India is in May.

So, Statement 2 is not correct.

It occurs not only in the inter-continental regions in India but also occurs in the coastal state
ssuch as West Bengal, Odisha, Gujarat, parts of Maharashtra, Karnataka, Andhra Pradesh,
and Telangana where the actual maximum temperature is 37°C or more. Sometimes it occurs
in Tamilnadu and Kerala also.

So, Statement 3 is not correct.

Q.84) Crop diversification in agriculture can be used as a tool to

1. Promote sustainable agriculture


2. Reduce Import dependence
3. Higher incomes for the farmers
4. Bring in water use efficiency and soil health sustainability

Select the correct answer using the code given below.

(a) 3 only
(b) 1 and 3 only
(c) 2 and 3 only
(d) 1, 2, 3 and 4

EXPLANATION:

Diversification of cropping and farming systems is a central agroecological principle, which may
improve resource use efficiency, reduce pests and diseases, enhance the resilience of the
production by promoting sustainable agriculture and manage the resources, soil quality and
satisfy present generation needs without compromising future generation needs.

Diversification of crops helps a country to grow all the required crops for its consumption inside
its own territory without depending on the imports from other country and reduce the country‘s
dependence on imports.

By diversifying the crops, a farmer would have many varieties of crops to sell and increase his
income.

71
Different crops consume different water quantities and different minerals from soil so
diversifying crops helps to ensure water use efficiency and soil health sustainability.

So, Option (d) is correct.

Q.85) ―Cane reservation area‖, sometimes seen in the news recently, is related to which one of the
following?

(a) Providing price assurance to the climate resilient areas


(b) Restricting the sugarcane production to selected areas in a district
(c) Growing sugarcane in a particular area for ethanol production
(d) Purchase of sugarcane from farmers within a specified radius

EXPLANATION:

The cane reservation area mandates cane farmers to supply their sugarcane to the specific sugar
mill. Every designated mill is obligated to purchase from cane farmers within the cane
reservation area, and conversely, farmers are bound to sell to the mill. This ensures a minimum
supply of cane to a mill while committing the mill to procure at a minimum price. So, Option (d)
is correct.

Q.86) Which one of the following statements best describes the term ‗‘Imported Inflation‖?

(a) Inflation due to higher levels of external debt


(b) Inflation due to excess flow of foreign investments.
(c) Inflation due to elevated global energy prices
(d) Inflation due to international sanctions

EXPLANATION:

When the general price level rises in a country because of the rise in prices of imported
commodities is called Imported Inflation. For example, a global increase in the price of crude oil
(energy), gold, etc will result in inflation in that particular country due to its imports. So, Option
(c) is correct.

Q.87) Consider the following statements about theMinimum Support Price (MSP) in India:

1. The expected return to farmers over the cost of production is uniform across all crops covered
under MSP.
2. The differential minimum support price provided to various crops may help in reducing the
baseline water stress indirectly.
3. MSP for Toria and de-husked Coconut are determined based onSunflower and Copra respectively.

72
Select the correct answer using the codes given below

(a) 1 only
(b) 2 only
(c) 1 and 3 only
(d) 1,2 and 3

EXPLANATION:

The expected return to farmers over the cost of production is different across all crops covered
under MSP. It is estimated to be highest in the case of Wheat (100%) and rapeseed/mustard
(100%), followed by lentil (79%) and gram (74%); barley (60%); safflower (50%).

So, Statement 1 is not correct.

The different remuneration is given to various crops which aims at crop diversification which
may either increase the water stress or reduce the water stress based on the crop which is
chosen for cultivation. If cereals, pulses are grown then it may reduce water stress by giving
more remuneration to these crops it may promote the cultivation of these crops andreduce the
water stress indirectly.

So, Statement 2 is correct.

The Government of India announces Minimum Support Prices (MSP) for major agricultural
commodities of Fair Average Quality (FAQ) each year in both the Crop seasons after taking into
account the recommendations of the Commission for Agricultural Costs and Prices (CACP). In
addition, MSP for Toria and de-husked coconut is also fixed based on MSPs of rapeseed &
mustard, and copra respectively. So, Statement 3 is not correct.

Q.88) Which of the following factor(s) causes the Western Disturbances in the northern parts of India?

1. Tropical Westerly Jet stream blowing in the south of Pamir Knot.


2. Alignment of Mountains in South West Asia.
3. Intense Heating of North Indian Plains.

Select the correct answer using the code given below.

(a) 1 only
(b) 1 and 2 only
(c) 2 and 3 only
(d) 1, 2 and 3

73
EXPLANATION:

A western disturbance is an extratropical storm originating in the Mediterranean region that


brings sudden winter rain to the northern parts of the Indian subcontinent.

Pamir knot is a place where several south-central Asian mountain ranges radiate.This sub-
tropical westerly Jetstreams gradually travel across the middle-east from Iran, Afghanistan,
and Pakistan south of Pamir knot ranges to enter the Indian sub-continent.So, Statement 1
is correct

As the Mountain ranges in Southwest Asia are Parallel to the Sub-tropical wind flowing from
the Mediterranean sea, it allows the westerly jetstreams to pass through it without blocking
them. So, Statement 2 is correct.

74
Since the Western Disturbance originates in the Mediterranean Sea, as a low-pressure area
or a trough over the surface or the upperair in the westerly winds regime, north of 20
degrees causes changes in pressure, wind pattern, and Temperature fields. It is accompanied
by cloudiness, with or without precipitation.Intense heating of the Northern Indian plain
does not cause western disturbance in India. So, Statement 3 is not correct.

Q.89) When the Russian war ships, stationed at the Black Sea, need to reach the Mediterranean Sea in
the shortest possible route, then the ships must cross which of the following water bodies?

1. Sea of Azov
2. Marmara Sea
3. Aegean Sea

Select the correct answer using the code given below.

(a) 1 only
(b) 1 and 2 only
(c) 2 and 3 only
(d) 1, 2 and 3

EXPLANATION:

To reach the Mediterranean sea from the Black sea, the Russian warship need not go to the
Sea of Azov which lies in its north. Rather they need to go from the Black Sea to the Sea of
Marmara to its Southwest then to the Aegean Sea and finally to the Mediterranean Sea. So,
Option (c) is correct.

75
Q.90) Consider the following cities:

1. Kanthkot of Gujarat
2. Udaipur of Tripura
3. Champhai of Mizoram
4. Kalinjarh of Rajasthan

Identify the commonality among the cities given above.

(a) These cities have a high diurnal range of temperature


(b) The Tropic of Cancer passes through these cities
(c) These are the cities that have the higher rainfall variability in India
(d) These are the newly declared wetlands under the Ramsar Convention

EXPLANATION:

The Tropic of Cancer passes through eight states in India:

 Gujarat (Jasdan, Kanthkot)


 Rajasthan (Kalinjarh)

76
 Madhya Pradesh (Shajapur)
 Chhattisgarh (Sonhat)
 Jharkhand (Lohardaga)
 West Bengal (Krishnanagar)
 Tripura (Udaipur)
 Mizoram (Champhai)

So, Option (b) is correct

Q.91) Consider the following statements regarding the Prayag Assembly of Harsha:

1. Fahien mentions in his account about the conference held at Allahabad, known as Prayag.
2. It was one among the conferences routinely convened by Harsha once in five years.
3. In these conferences, Harsha distributed his enormous wealth as gifts to the members of only
Buddhist religious sects.

Which of the statements given above is/are not correct?

(a) 1 only
(b) 1 and 2 only
77
(c) 2 only
(d) 1 and 3 only

EXPLANATION:

Hiuen Tsang mentions in his account about the conference held at Allahabad, known as Prayag. It
was one among the conferences routinely convened by Harsha once in five years. So, Statement 1
is not correct and Statement 2 is correct.
In these conferences, Harsha distributed his enormous wealth as gifts to the members of all
religious sects. According to Hiuen Tsang, Harsha was so lavish that he emptied the treasury and
even gave away the clothes and jewels he was wearing. So, Statement 3 is not correct.

Q.92) Consider the following statements regarding the market reforms of Alauddin Khalji:

1. Alauddin Khalji fixed the prices of all commodities from grain to cloths, slaves and cattle.
2. A controller of the market (shahna-i-mandi), intelligence officers (barids) and secret spies
(munhiyan) were appointed.
3. The grain merchants were placed under the Shahna-i-mandi and regrating or ihtikar was
prohibited.

Which of the statements given above is/are correct?

(a) 1 only
(b) 1 and 3 only
(c) 3 only
(d) 1, 2 and 3

EXPLANATION:

In the early 14th century, the Delhi Sultanate ruler Alauddin Khalji instituted price controls and
related reforms in his empire. He fixed the prices for a wide range of goods, including grains,
cloth, slaves, and animals. So, Statement 1 is correct.
He appointed the controller of the market who is called shahna-i-mandi, intelligence officers
called barids, and secret spies called munhiyan. The grain merchants were placed under the
Shahna-i-mandi and he also prohibited the ihtikar system (sale and purchase transactions). So,
Statements 2 and 3 are correct.

Q.93) Consider the following pairs regarding the Maratha Ministry

Designation Offices

1. Peshwa - Military Commander


2. Amatya - Civil servants
3. Nyayadhish - Chief Justice

78
Which of the pairs given above is/are correctly matched?

(a) 1 only
(b) 1 and 2 only
(c) 2 and 3 only
(d) 3 only

EXPLANATION:

Under the Maratha administration, the ruler Sivaji was assisted by the council of ministers
called Ashtapradan.

 Peshwa - Prime Minister


 Amatya - Finance Minister
 Sar-i-Naubat or Senapati - Commander in Chief
 Waqia Navish - Managing interior affairs
 Sumant – Master of Ceremonies and Foreign Affairs minister
 Nyayadhish – Administration of Justice
 Sachiv – Royal Correspondence
 Panditrao – Religious administration

So, Option (d) is correct.

Q.94) Consider the following statements regarding the Coastal ecosystems:

1. The coastal ecosystems of mangroves, tidal marshes and seagrass meadows contain more soil
organic carbon and less living biomass than Tropical forests.
2. Unlike terrestrial ecosystems, coastal carbon stocks are not accounted for under UNFCCC
guidelines and national GHG inventories.
3. To protect the Coastal ecosystem, the Indian government has created a Marine Protected Area
(MPA) and Coastal Regulation Zone (CRZ) under Environment Protection Act 1986.

Which of the statements given above is/are correct?

(a) 1 only
(b) 1 and 2 only
(c) 2 and 3 only
(d) 2 only

EXPLANATION:

Thecoastal ecosystems of mangroves, tidal marshes, and seagrass meadows contain large stores of
carbon deposited by vegetation and various natural processes over centuries. It contains more soil
organic carbon and living biomass when compared to Tropical forests. So, Statement 1 is not
correct.

79
Unlike terrestrial ecosystems, coastal carbon stocks are not accounted for under UNFCCC guidelines
and national GHG inventories as of now. But, presently talks are going on the global forum to consider
accounting for the coastal carbon stocks. So, Statement 2 is correct.
To protect the Coastal ecosystem, the Indian government has created a Marine Protected Area
(MPA)which is essentially a space in the ocean where human activities are more strictly regulated than
the surrounding waters - similar to parks we have on land. These places are given special protections
for natural or historic marine resources by local, state, territorial, native, regional, or national
authorities under the wildlife protection act of 1972 and the Coastal Regulation Zone (CRZ) is formed
under the Environment Protection Act 1986. So, Statement 3 is not correct.

Q.95) Which one of the following pair is the predominant species that exist in the Desert National Park in
Rajasthan?

(a) Bengal Tiger, Great Indian Bustard, Chinkara, Indian Wolf

(b) Great Indian Bustard, Chinkara, Indian Wolf, Bengal Fox

(c) Bengal Tiger, Great Indian Bustard, Chinkara, Desert Cat

(d) Bengal Tiger, Great Indian Bustard, Chinkara, Monitor Lizard

EXPLANATION:

Desert National Park is a national park situated in the Indian state of Rajasthan. This is one of the
largest national parks, covering an area of 3162 km². The Desert National Park is an excellent
example of the ecosystem of the Thar Desert. The predominant species that exist in the Desert
National Park in Rajasthan are Great Indian Bustard, Desert Fox, Bengal Fox, Indian Wolf, Monitor
Lizard, Chinkara, etc. So, Option (b) is correct.
The Bengal Tiger is a highly adaptable animal and lives in a wide range of different habitats. These

80
include forests, mangroves and wetlands. They cannot be found in desert regions. So, other options
can be eliminated.

Q.96) Which one of the following pair of countries shares its border with Russia through both land and
sea?

(a) Norway, Finland, Japan and the United States of America

(b) Norway, Finland, Estonia, Poland and Ukraine

(c) China, Mongolia, Latvia and North Korea

(d) Latvia, Lithuania, Estonia and Belarus

EXPLANATION:

Russia shares land borders with Norway and Finland to the northwest; Estonia, Latvia, Belarus,
and Ukraine to the west, as well as Lithuania and Poland (with Kaliningrad Oblast); Georgia
and Azerbaijan to the southwest; Kazakhstan and Mongolia to the south; China and North
Korea to the southeast — while having maritime boundaries with Japan and the United States.
So, Option (b) is correct.

81
Q.97) Consider the following statements regarding the Forest conservation Act of 1980.

1. This Act made it necessary to form an Advisory Committee by the respective State government for
diverting any forest land for ―non-forestry purposes‖.
82
2. This Act prohibits the State government to clear the naturally grown trees in that land or portion,
for the purpose of using it for reafforestation.

3. This Act prohibits the State government from diverting the forest land for the purpose of the
cultivation of tea, coffee, spices, rubber, palms, oil-bearing plants, horticultural crops or medicinal
plants.

Which of the statements given above is/are correct?

(a) 2 only

(b) 1 and 2 only

(c) 2 and 3 only

(d) 3 only

EXPLANATION:

According to the Forest Conservation Act of 1980, The Central Government may constitute an
Advisory Committee consisting of such number of persons as it may deem fit to advise that
Government with regard to
(i) the grant of approvalof forests or use of forest land for non-forest purposes and
(ii) any other matter connected with the conservation of forests which may be referred to it by
the Central Government. So, Statement 1 is not correct.

The Act puts a Restriction on the dereservation of forests or use of forest land for a non-forest
purpose like any forest land or any portion thereof may be cleared of trees which have grown
naturally in that land or portion, for the purpose of using it for reafforestation.

The main aim of the act is to protect the forests and prevent deforestation that will lead to land
erosion and subsequent degradation of the land.Clear the naturally grown trees in that land or
portion of it for reafforestation purposes will destroy the main objective of the Act.So,
Statement 2 is not correct.

According to the Act,no State Government or other authority shall make, except with the prior
approval of the Central Government, any order directing that any forest land or any portion
thereof may be used for any non-forest purpose.
―non-forest purpose‖ means the breaking up or clearing of any forest land or portion thereof for
any purpose for,
 the cultivation of tea, coffee, spices, rubber, palms, oil-bearing plants, horticultural crops
or medicinal plants
 any purpose other than reafforestation,
So, Statement 3 is correct.

83
Q.98) Which of the following are the important objectives of the National Mission for Clean Ganga?

1. It aims to ensure effective abatement of pollution and rejuvenation of the river Ganga.

2. It aims to maintain minimum ecological flows in the river Ganga to ensure water quality.

3. It aims to preserve and rear the Gangetic dolphin in its natural habitat.

Select the correct answer using the code given below.

(a) 1 and 3 only

(b) 1 and 2 only

(c) 2 and 3 only

(d) 1, 2 and 3

EXPLANATION:

In 2011, the National Mission for Clean Ganga (NMCG) was listed as a society under the Societies
Registration Act, 1860. The consortium acts as the implementation arm of the National Ganga
River Basin Authority (NGRBA), which was constituted under provisions of the Environment
Protection Act (EPA), 1986, and was established to address pollution challenges in the Ganga
river.

The main objectives of the NMCG are as follows,

 To ensure effective abatement of pollution and rejuvenation of the river Ganga.


 It aims to maintain minimum ecological flows in the river Ganga to ensure water quality
 It also undertaking massive afforestation activities to conserve biodiversity, along with
developing the flora & fauna and carrying out effective plans & policies to conservedolphins,
turtles, and varieties of fish.

So, Option (d) is correct.

Q.99) Consider the following statements regarding the Carbon Border Adjustment Mechanism, recently
seen in the news.

1. It is purported to be an improvement on the Emissions Trading System under the Glasgow Climate
Summit of 2021.
2. It proposes to reduce the risk of carbon leakage and level the field for the industries working to
decarbonize their production processes.
3. It caps the Green House Gas emissions that can be released by certain industries.
4. India and other developing countries wholeheartedly support this mechanism as it favours the
developing countries.

Which of the statements given above are correct?

(a) 1 and 4 only


84
(b) 2 and 3 only
(c) 1, 2 and 3 only
(d) 1, 2, 3 and 4

EXPLANATION:

Carbon Border Adjustment Mechanism(CBAM) is purported to be an improvement on the


Emissions Trading System (ETS) of the European Union (EU). ETS is the world‘s first
international emissions trading scheme and the EU‘s flagship policy to combat climate change,
but it was launched in 2005 and not at the Glasgow climate summit in 2011. So, Statement 1
is not correct.

The CBAM proposes to reduce the risk of carbon leakage and to level the field for EU industries
working to decarbonize their production processes. This will be done by imposing a carbon tariff
on carbon-intensive products such as cement and electricity, imported by the EU. So,
Statement 2 is correct.

It sets a cap on the amount of Greenhouse Gas (GHG) emissions that can be released by certain
industries. It also addresses leakage, but it also dampens the incentive to invest in greener
production at home and abroad. So, Statement 3 is correct.

India and other developing countries did not even support this mechanism due to its impact

on their export and competitiveness. So, Statement 4 is not correct.

Q.100) Consider the following statements regarding the Bio-Mining of solid waste:

1. In the Bio-mining process, the garbage is treated with bio-organisms or natural elements like air,
sunlight, etc.

2. It gives efficient results even in the old and aged landfills where the waste has already reached the
maximum level of microbial degradation.

3. Bioremediation is only possible for dumpsites having a higher organic content.

Which of the statements given above is/are correct?

(a) 1 only

(b) 2 only

(c) 2 and 3

(d) 1 and 3

85
EXPLANATION:

Bio-mining is a technically assisted and economically managed extraction of recyclables and other
revenue-generating fractions from waste materials already been disposed of by landfilling. In the
Bio-mining/ Landfill mining process, the garbage is treated with bio-organisms or natural
elements like air, sunlight, etc Bio-mining of dumpsites is aided by the bioremediation process. So,
Statement 1 is correct.

Bioremediation is only possible for dumpsites having a higher organic content (such as sites where
fresh waste is disposed of along with legacy waste). Bioremediation of legacy waste does not
necessarily give efficient results in old/aged landfills (un-operated and closed dumpsites) where
the waste has already reached the maximum level of microbial degradation. Bioremediation is a
microbe-mediated degradation of organic waste carried out by adding biological inoculum to the
dumpsite. So, Statement 2 is not correct and Statement 3 is correct.

86

You might also like